Will the real god God please stand up?

[Updated Dec. 28, 2017, with a revised and expanded discussion of arguments from Sean Carroll's book The Big Picture. - SC]

There are many reasons why I’m not retired, but one of the bigger ones is that I haven’t figured out yet how to get at least a quarter (if not a dollar bill) from every person who’s ever asked me how I can believe in “a god or gods” in an age of “science” and “reason”. The question is usually sincere rather than an attempt to troll, but either way, the wording alone is enough to reveal where things are headed, and the ensuing discussions have been nothing if not utterly predictable. In virtually every case the underlying narrative was based on the same handful of fashionable just-so stories, none of which appeared to have ever been questioned.

Back in days of yore, I was told, bucolic ancients looked out on a universe resplendent with mysteries they could neither understand nor predict, yet depended on for their survival. For all its dependable seasons and regularities, the universe visited floods, fires, and other tragedies on them as often as it yielded its bounty. In their attempt to understand why and find a just order to it all, they attributed these mysteries to the capricious activities of spirits called “gods” who were like us in every respect, except that they were disembodied and endowed with vast magical powers over various parts of the natural order. As the rise of science rolled back these mysteries with rational explanations, such gods were no longer needed to account for them. Eventually, the faiths based on them were rendered superfluous, and thus did Science triumph over religion (note the capital “S” and lower-case “r”).

There are so many things wrong with this it’s difficult to know where to begin. Perhaps the best way to unpack this mess is to start with the origins of the God of Classical Theism on which the Abrahamic religions are founded. These cover the professed religious beliefs of well over half of humanity and roughly 80% of North America and account for virtually every instance of the above narrative I’ve ever personally witnessed.1

Contrary to widespread belief, Classical Theism as a formal system of thought didn’t originate with Christianity or Judaism, nor was it an attempt to explain any mystery of the natural world (which makes it quite telling that the God that eventually emerged from that tradition bore a striking similarity to the uniquely monotheistic God of the Old Testament that the Israelites had been worshipping via revelation for nearly a millennium). The seminal theological question never was “is there a god?”—it is, and always has been, “why is there something rather than nothing?” In the Fifth Century BC, the Greek philosopher Parmenides formulated an axiom that was later Latinized as ex nihilo nihil fit (“out of nothing comes nothing”). Unless you believe in magic this is as straightforward as axioms get, and for nearly 2500 years no thinker of any repute has seriously challenged it. [At least not until the present day, when a handful of metaphysically illiterate Atheist physicists decided that philosophy is “dead” because it hasn’t kept up with their profession, and gave themselves permission to redefine the word “nothing” and make Magic a sub-discipline of physics. But that’s a topic for another day.] This, in turn, raised other issues. Parmenides went on to argue that change and differentiation must be illusory, for to change, he said, is for something to cease to exist in one state and begin to exist in another. Because that would require things to come from nothing, and disappear back into it, he considered it absurd. And yet, change is every bit as indisputable a fact of life as existence itself. What are we to make of these two realities, and how they relate to each other? For the next one or two centuries, philosophers of different schools argued these questions, some emphasizing the primacy of change, and others the primacy of the unchanging unity of things.

The first true leap forward came circa the mid-Fourth Century BC when Aristotle published his Metaphysics. Aristotle argued that the apparent tension between being and becoming can be accounted for if we differentiate between the actual state of existence of real-world things (or substances) and their innate potentialities for existing in different ones (later Scholastic thinkers denoted these respectively as acts and potencies). Change occurs when the active potencies of one substance causally instantiate outcomes from the passive potencies of another via four types of causality—Their material constituents (material causality), their essential form and identifying properties (formal causality), their direct physical interactions (efficient causality), and their directedness toward ends (final causality). For instance, we could say that the motion of massive objects reflects their mass and other properties (material and formal causes), and the forces they interact with (efficient causes). Aristotle would also say that they fall to the ground when dropped because the earth is their natural resting place (final causality). Similar ideas were developed by Plato, and by the Stoics and Neoplatonists after him, and eventually brought to fruition by medieval Scholastic philosophers and theologians of the Christian, Jewish, and Islamic traditions. Various schools of thought were represented in each, but most if not all, eventually converged on some combination of the following axioms;

1)   The universe is contingent. Its essential nature, or form (and that of everything in it) is separate from its existence. [e.g. - We can meaningfully conceptualize horses and unicorns without regard to whether there are any.]

2)   The universe is causally interconnected. The acts and potencies of its physical constituents are interrelated in rationally consistent ways.

3)   The universe evolves. Per 2), its actual state of existence changes from moment to moment in dependable ways. [e.g. - Seeds grow into trees, objects fall toward a gravitational source, etc.] As such, science is a meaningful endeavor that gives us real, grounded knowledge about the way the world is.

4)   Potencies may be active powers or passive capacities for change, and the events that unfold from their activity may be (formal terms again) essentially ordered, or accidentally ordered (dependent on, or independent of the continuing activity of their cause/s). [e.g. - A father has the active power to father children, and his kids will continue to exist whether he continues fathering behavior or not (accidentally ordered events). A guitar has the passive power to make music by actualizing the passive power of air to produce sound, but only if it is played by a musician, and the music will exist only while the guitar is being played (essentially ordered events).]

5)   Purely passive potentialities cannot self-realize—they must be instantiated (made actual) by something else that is actual. [e.g. - wood has the passive potentiality to burn, but only if it's exposed to an actual source of heat. An infinitely long chain of stationary railroad cars (or one connected in a loop) cannot move, even though each car is connected to one that can pull it. There must be a least one engine with the active potency for inducing motion.]

6)   The universe's actualities and potentialities are a mix of active powers and passive possibilities. [e.g. - A locomotive has the active power to pull a train of cars with passive potentials for motion, but also has other passive dependencies, such as the need for an engineer; you have the active power to walk or run, but not to continue living without food and water; etc.]

7)   As persons with active and passive potencies of our own, we are rational, freely choosing, intentional agents. As such, our observations and thoughts can, and do, give us reliable knowledge of the universe.

From these (particularly the concept of essentially-ordered causality), they concluded that there must exist something that is pure act—the ground of all being and empowered possibility, with no passive potentialities or dependencies (Davies, 2004; Feser, 2010; 2014). Furthermore, this pure act must be;

a)   Eternal - Not within, or in any way constrained by time or space.

b)   Unchanging – Not evolving per any passive potencies susceptible to influences external to itself.

c)   Simple - A substantial, or essential unity without parts or differing properties of the sort possessed by physical things.

d)   Omnipotent - Unlimited in active powers.

e)   Omniscient - Present in, and aware of, all that is.

f)   Possessing both intellect and will, and as such, is the ground of all personhood (as opposed to being "a" person).

g)   The intentional cause of everything else that is, and thus, the objective source of the meaning, value, and purpose of things.

Aristotle referred to this pure act as the Unmoved Mover. Christian, Jewish, and Islamic philosophers recognized Him as the God of Classical Theism who appears in the Bible and Quran. How these conclusions were reached, and how this timeless, changeless God is related to the Christian Trinity and His portrayal in the pages of both Scriptures, would fill numerous posts and is beyond our scope today. But before we proceed, a few comments are in order.

First, it’s widely believed that Aristotle’s metaphysics is dependent on his outdated physics, and therefore no longer relevant today. In his 2014 debate with William Lane Craig, Atheist physicist Sean Carroll spoke for many when he addressed transcendent causality and the universe (Carroll & Craig, 2014) stating that,

“[T]here’s a bigger problem with it, which is that it is not even false. The real problem is that these are not the right vocabulary words to be using when we discuss fundamental physics and cosmology. This kind of Aristotelian analysis of causation was cutting edge stuff 2,500 years ago. Today we know better. Our metaphysics must follow our physics. That’s what the word metaphysics means...

[T]he way physics is known to work these days is in terms of patterns, unbreakable rules, laws of nature... There is no need for any extra metaphysical baggage, like transcendent causes, on top of that. It’s precisely the wrong way to think about how the fundamental reality works.”

All of this is either false or grossly misleading. In modern analytic philosophy, Aristotelian/Scholastic concepts of ontology and causality are every bit as active a field of study as they’ve ever been (e.g. - Martin, 1997; Davies, 2004; Feser, 2014; 2015; Oderberg, 2008, etc. and sources cited therein). There are, of course, differing schools of thought on them, and their relationship to the sciences is actively debated. Some lean toward a deep interrelationship between physics and these metaphysical ideas. Others such as Edward Feser (2010; 2014; 2015) argue that the two are entirely separate realms. Aron and I fall somewhere in the middle. [For more, see Aron’s entire series of posts on Fundamental Reality.]

While it is true that modern physics treats causality differently than Aristotle and the Scholastics did (e.g. - the notions of material and formal causes are largely redundant in physics and not really needed), clearly the two realms of thought speak to the same underlying realities and even share some common language. The very “patterns, unbreakable rules, laws of nature” Carroll speaks of inherently imply an underlying unity which not only makes physics possible but fits the terms act and potency beautifully. Potentials, for instance, are a regularly recurring theme in physics, and the fact that equations of motion can be derived from them also bears a striking similarity to the Aristotelian notion of final causality. The dynamics of a falling mass can be differentially specified in terms of a static gravitational potential, but a Scholastic would say that the mass falls to earth because that’s its natural resting place. The ideas being expressed here aren’t as different as many suppose. Another common misconception is that final causality involves teleology. In fact, it’s about directedness as much as purpose or design, if not more, and applies to inanimate objects as well as living things. It’s not a huge leap to see directedness in the way static potentials lead to equations of motion.

These Aristotelian concepts are less rigorously developed of course, but conceptually at least, they substantially overlap their counterparts in physics, which implies at least some unity between the two. But at the same time, as we saw in my last post, the fact that there are numerous ontic interpretations of QM alone should give us pause before assuming that one of these realms is entirely supervenient on the other. In any event, wherever one falls on this spectrum, the one thing that isn’t true is that "our metaphysics must follow our physics". Nor is that “what the word metaphysics means" as Carroll claims. Aristotle’s Metaphysics was so named because he wrote that book after he wrote his Physics, not because the former is in any less foundational than the latter, or entirely supervenient on it (in Greek, the root meta is equivalent to the Latin post, meaning “after”).

Second, it’s worth noting that this argument, which is known as the cosmological argument, is widely misunderstood. In popular writings, particularly those of its critics, it’s almost always presented as an argument for a historical creation event based on accidentally-ordered temporal chains of causality when in fact, it’s based entirely on essentially-ordered, or simultaneous causality.2 The traditional example given by St. Thomas Aquinas and other Scholastics is that of someone pushing a ball with a stick. The passive potency of the ball for rolling motion is realized only while it is being pushed by the stick’s passive potency for doing so, which in turn is realized only while the one wielding it is exercising his/her active potency for wielding it to push objects. The entire causal chain is simultaneous in the present moment and has nothing whatsoever to do with any cause or causes that may have existed even a few seconds prior. In fact, Aquinas, who developed the argument better than anyone else in history, famously believed that it wasn’t possible to demonstrate that the universe had a temporally-ordered causal beginning. He believed it did because Scripture said so, but he felt that observation and philosophical arguments alone couldn’t demonstrate that. Today, of course, Carroll’s dismissal of transcendent causes notwithstanding, the evidence for a beginning is considerable and whether they admit it or not, a source of dismay for Atheists. Aquinas’ claims to the contrary are relevant here, only to the extent that they emphasize that time-ordered causality plays no role in traditional cosmological arguments.

Furthermore, in the writings of Aristotle and the Scholastics, the term move denotes change in general, not just rectilinear motion as we understand it. To them, changes in any property—including say, color, temperature, or even a beginning of existence—would be considered “movement”. Interestingly, Carroll misses the subtleties of this as well. In his book The Big Picture (2017) he tells us that,

"[T]he whole structure of Aristotle's argument for an unmoved mover rests on his idea that motions require causes. Once we know about conservation of momentum, this idea loses its steam... What matters is that the new physics of Galileo and his friends implied an entirely new ontology, a deep shift in how we thought about the nature of reality. 'Causes' didn't have the central role that they once did. The universe doesn't need a push; it can just keep going." (My emphasis)

Clearly, this argument doesn’t account for accelerated motion, which anyone who’s ever dropped a $600 cell phone off a balcony will tell you, is quite real. For some reason, this doesn’t seem to concern him. The real puzzle, however, is that he acknowledges that Aristotelian motion is a much broader concept than mere spatial displacement, and even uses the word transformation to describe it. Why he imagines that an argument against an untransformed transformer could be based on rectilinear motion alone is anyone’s guess. The metaphysical importance of conservation of momentum, he tells us, is “hard to overemphasize” and he sees in it an underlying principle that in his view, can be extrapolated to all contingency and change. But how this is supposed to work in practice is never clarified. Throughout this chapter (aptly titled The World Moves by Itself) he speaks of “causes” and “motions“ in the most general metaphysical sense and uses those terms interchangeably. But the only working examples he offers involve frictionless displacement of objects like coffee cups, which he supplements with glib remarks about how terms like “cause” and “effect” aren’t found in physics textbooks (as though the language of physics and its methods are the only ones that are meaningful in the real world).

Near as I can tell, Carroll believes that conservation of momentum is built on a metaphysical foundation that generalizes to all conservation laws. Essentially, this amounts to the claim that Noether’s theorem (and possibly its extension to quantum field symmetries) constitutes a sort of “blood-brain barrier” isolating all contingent change in the universe from the interventions of any creator god. If so, the problems with this are obvious. For starters, he points out (correctly) that Aristotle’s unmoved mover was later fully developed by Aquinas. As we’ve already seen, essentially-ordered causality and God as the universe’s sustainer as well as its creator are foundational concepts in his thought. Anyone even remotely familiar with this will immediately recognize a universe that “keeps going” after an initial “push” as one based on an independent temporally-ordered causal chain that some divine machinist occasionally tinkers with—an argument that Aquinas went to great lengths to refute, and clearly not the cosmological argument he defended. Second, attributing virtually all contingency and change to conservation laws is, to say the least, a stretch. What sort of conservation law gave me blue rather than brown eyes, for instance, or required me to order a triple-shot cappuccino this morning rather than a hot chocolate? Even if we ignore all this, there’s one rather large elephant in the room that isn’t being addressed. The sort of conservation laws Carroll is appealing to are only valid over locally flat regions of space-time. For the universe as a whole, neither momentum nor energy is even well-defined, much less conserved (MTW, 2017)—a fact that he’s not only aware of but has written about elsewhere himself (Carroll, 2010), yet now conveniently chooses to forget.3

It’s odd that Carroll manages to muddle so many metaphysical concepts as completely, and chronically, as he does. Unlike many scientists these days, he has a background in philosophy (having minored in it as an undergraduate) and is known for his thoughtfulness and attention to detail with metaphysical topics. He’s repeatedly, and rightly, called out many of his colleagues for their Philistine recklessness in these areas and with philosophy in general. If anyone should know better, it would be him.

Finally, it should also be noted that the history of thought on God’s nature isn’t quite as monolithic as I perhaps made it sound. In recent years, for instance, some theologians and philosophers of religion have questioned the notions of God as grounded personhood (as opposed to personality), His simplicity, and the claim that He’s timeless and unchanging. God, it’s argued, cannot be meaningfully omniscient and loving, as He’s presented in the Bible and Quran, unless He has attributes that manifest in a personality, not unlike ours, and He in some sense experiences time (although opinions as to whether His time maps onto the space-time of our experience, and if so, how). This school of thought, referred to by some as theistic personalism, has been particularly popular among advocates of presentism (the so-called “A-Theory” of time). It’s more notable advocates include Richard Swineburne, Alvin Plantinga, J.P. Moreland, and William Lane Craig.

Theistic personalism is a relatively late development in the history of Classical Theism and hasn’t gained widespread acceptance among theologians and philosophers of religion (Davies, 2004). The traditional arguments for the simplicity and timelessness of the God of Classical Theism as presented above are formidable and well-supported not only by metaphysics but the Abrahamic Scriptures as well. The apparent difficulties presented by a timeless God in changing history are not as difficult as they may seem at first blush either. Once we realize that if God is omnipresent throughout His created space-time, and interacting with it at every point according to His Will, He will appear to change from the standpoint of time-bound creatures like us, much the way a static landscape appears to change to the passengers of a car driving through it. Dispensing with all this simply to bring God more in line with our experience adds layers of arbitrary, and unnecessary metaphysical complexity that cry out for Occam’s Razor. As if that weren’t enough, it runs badly afoul of physics as well. The presentism that it most naturally fits has numerous issues, not the least of which are the difficulties of reconciling it with the Lorentz boost. While it is possible to make presentism work in a relativistic framework (Copan & Craig, 2004), the match ain’t exactly made in Heaven and IMHO at least, creates far more problems than it solves. Nevertheless, theistic personalism does have its place in modern theological discourse, and it has been ably defended by its proponents (Moreland & Craig, 2003).

There… Now that all the fine print is out of the way, let’s return to our seven-axiom argument for the existence of God. At this point, several things should be readily apparent.

1)   God is not “a god”

When Atheists (or more commonly, New Atheists) speak of "a god or gods" what they invariably have in mind are demigods—minor deities of the sort one finds in ancient mythologies. These are the disembodied space and time-bound magical spirits central to their narrative. In The God Delusion Richard Dawkins (2008) states that,

"I have found it an amusing strategy, when asked whether I am an atheist, to point out that the questioner is also an atheist when considering Zeus, Apollo, Amon Ra, Mithras, Baal, Thor, Wotan, the Golden Calf and the Flying Spaghetti Monster. I just go one god further."

The problem with this is obvious—the “gods” he names bear no resemblance whatsoever to the God of Classical Theism. In Greek mythology, Zeus had a family tree like us. He was the child of the Titans Chronos and Rhea, and they were, in turn, descended from the primordial Greek deities (Wikipedia, 2016). Like the rest of the Greek pantheon, not only was he a time-bound spirit, he was earth-bound as well and "lived" at a physical location (Mt. Olympus). In fact, as often as not, such demigods were deified human rulers. Case in point, the Akkadian ruler/gods Gilgamesh and Naram-Sin who respectively ruled during the late Third and early Second Millennia BC (Armstrong, 2015).

God on the other hand (note the capital “G”), is the ground of all being and personhood. He is neither space and time-bound nor an instantiation—there is no general class of things called "grounds of all being" of which He can be said to be one example among many. The very claim that there could be more than one such ground is inherently self-contradictory. It’s no accident that the Abrahamic religions are all monotheistic. And as the creator of all else that exists—including the very space-time manifold whose geometry is, per general relativity, related to the mass-energy and momentum it contains—calling Him a demigod amounts to claiming that He's bound by His own creation, and dependent on it for His existence. That, my friends, is patently absurd. Saying that God is "a god” isn't merely wrong, it's a category error.

Interestingly, the distinction we find today between the anthropomorphic personified God of televangelist’s sermons and children’s picture Bibles, and the God of Classical Theism was every bit as true in Aristotle’s day as well. Then, as now, philosophers distinguished between Everyman’s bearded, gray haired Zeus who threw thunderbolts from Mt. Olympus, and the classical theistic "Zeus" (or more properly, Greek primordial God) of formal thought. If this were the 4th Century BC, New Atheists like Dawkins would be out in front of the Athens Peripatetic school in togas beating their well-inflated chests about "a zeus or zeus'es," and Aristotle would be the one biting his tongue and doing whatever could be done to educate them. Some things never change… ;-)

2)   God is not a hypothesis

Science doesn’t deal in “facts” (at least not as most people understand that word). More correctly, it deals with data. One begins with reproducible measurements of some observed phenomena (e.g. – the power density spectrum of the cosmic microwave background, or tracks emerging from particle collisions in a cloud chamber). One or more hypotheses are formed to account for them, and the most viable of these are developed into formal theories from which the outcomes of further, yet untested observations can be predicted. In the case of physics, this generally means a set of differential equations and boundary conditions, a Lie algebra that embodies an expected symmetry, or the like. Failure of a theory’s predictions is its null hypothesis and counts as evidence against it. If further experiments yield the predicted outcomes, confidence in the theory grows, and if not, suspicion does. In this sense, hypotheses that make no testable predictions cannot meaningfully be called scientific.4

Enter our axioms 1) through 7). Though all are based on observation, and scientific illustrations could be given for them, they cannot be called “data” in any scientifically meaningful sense. How does one create a “dataset” to quantify concepts like act and potency, and use it to validate a ground of all being and personhood and the contingency of the universe? What they are, is a set of metaphysical axioms about the underlying ontic nature of the universe, and God (again, note the capital “G”) isn’t a hypothesis we postulate to account for them—He’s a formally reasoned conclusion derived from them.

Alright, before anyone blows a gasket, let me be clear about what I mean. No, I am not saying that the existence of God can be logically/mathematically proven. If it were that easy Atheism wouldn’t be a worldview worth discussing, and its proponents wouldn’t include some of the finest minds in history. What I am saying is that it’s a different sort of argument than the traditional data -> hypothesis -> test methodology science relies on. Claiming that there’s no evidence for God, as opposed to "a god or gods," is like claiming that there’s no “evidence” for “an equation or equations” called the Mean Value Theorem of Calculus. The Mean Value Theorem isn’t a hypothesis—it’s a formal proof that begins with certain axioms (e.g. – a continuous manifold, monotonic everywhere differentiable functions, etc.). The extent to which one accepts those axioms is the extent to which one accepts the conclusion. Likewise, to reject that conclusion is to reject the axioms it begins with.

Which brings us to the next point…

3)   Atheism is not a null hypothesis

Finally, we arrive at New Atheism's most beloved get-out-of-jail-free card—the belief that it's merely the rejection of Theism, and as such, a null hypothesis that needs no defense. Sam Harris (2008) minces no words when he states that,

“’Atheism’ is a term that should not even exist. No one ever needs to identify himself as a ‘non-astrologer’ or a ‘non-alchemist.” … Atheism is nothing more than the noises people make in the presence of unjustified religious beliefs.”

A New Atheist friend and colleague once put it to me even more starkly on social media,

“An Atheist is one who rejects the claims made by theists. An Atheist is simply a person who is not a theist. Atheism is not in itself a claim, and as such, simply cannot be false. Only claims can be proven false; a lack of claim cannot be said to be false. How can I be wrong when I say 'you haven't presented a compelling argument for your case'?” (My emphasis)

Clever, aren’t we? Don't state your claims directly, frame them as a rejection of someone else's… then conveniently excuse yourself from any responsibility for a proper defense of them, and set the standard of proof however high it needs to be to protect you, infinitely if necessary. Sleight of hand like this isn’t just bread-and-butter for New Atheists of course. Creationists and climate change skeptics rely heavily on it as well. Denial... it ain't just a river in Egypt anymore! ;-)

To be fair, this would be valid if we were postulating the activity of demigods in the created order as one possible explanation for some phenomenon. If my fishing buddy insists that the nibble I just had was a trout, I’m under no obligation to defend my skepticism when we both know the pond is full of bass and catfish as well. The burden of proof is on him to produce evidence for his “trout” theory as opposed to a bass or catfish one. But as we’ve seen, that’s not what’s happening here. We aren’t offering any “god hypothesis” to account for something in the natural world, whether it be trout in a pond or anything else. We’re formally demonstrating that a set of metaphysical axioms requires His existence. Atheists like Harris and my friend aren’t rejecting belief in “a god or gods”—they’re rejecting the metaphysical axioms that lead to the God of Classical Theism. That cannot be done in a vacuum without committing oneself to some, or all, of the following counter-axioms;

8)   The universe is a brute fact. Science may reveal its countless subtleties and underlying unities, but ultimately it just has the contingent features it does rather than an infinite number of other possibilities. There is no reason why... it just is that way.

9)   Per 8), the beginning of the universe's existence (13.73 billion years ago) is also a brute fact. There is no reason why... it just created itself from nothing.

10)   There is no such thing as causality—only events unfolding in certain ordered ways. “Causality” is just a concept we use to describe the appearance of mechanism between bits of stuff (what I referred to above as "interactions"), but ultimately those events are, to use David Hume's term, "loose and separate." They have no inherent relationship to each other.

11)   Matter does not actually possess any inherent properties or essential natures of the sort that could be described in terms of essence or potency (as I defined them above). Reality is ultimately just "bits of stuff" mechanically interacting according to mathematical laws expressed in terms of parameters that give the appearance of such. [“Um, ‘interactions’ and ‘laws’…? Didn’t you just say in 10) that…?” “Silence Dorothy! Pay no attention to that man behind the curtain...!”]

12)   The rationality of the laws of nature—that those "loose and separate" events between bits of stuff happen to unfold according to what physicist Eugene Wigner called "the unreasonable effectiveness of mathematics—is also a brute fact. There is no reason why... it just is that way.

13)   "Loose and separately" ordered bits of stuff are blind, and as such the universe ascribes no objective value or purpose. Everything in it, including us, is a byproduct of random, meaningless accidents—what Richard Dawkins called "blind, pitiless indifference" (Dawkins, 1996). Thus, morality is either nihilistic or entirely subjective.

14)   Alternately, if objectively normative moral values do exist—yours, mine, or anyone else's—then they too are brute facts. There is no reason why... they just are what they are. [“But my goodness gracious… isn’t it marvelous how nicely they align with mine…?”]

15)   Consciousness and personhood are illusory. To again use David Hume's term, we're just "bundles of percepts" in bodies made up of bits of stuff behaving according to deterministic laws. [“Um, ‘deterministic’…? Didn’t you say in 10) that…?” “Silence Dorothy! Pay no attention to that man behind the curtain...!”] "You" or "I" are concepts we use to describe our experience of the neural activity in our brains, and how it affects our perceptions and behaviors. Beyond that, we are no more “persons” in the sense of being freely empowered, intentional, and possessing rational agency than an email server is (analytic philosophers refer to this viewpoint as eliminative materialism).

16)   Though we are accidentally evolved "bundles of percepts," our perceptions and reasoned thoughts are reliable sources of knowledge of the deepest inner workings of the universe and ourselves.

Notice that these aren’t mere “rejections” of anything. Like 1) through 7), they’re positive metaphysical assertions about the ontic foundations of the universe, and as such, they have rational consequences. We can reject belief in mythological demigods, invisible dragons, or the Flying Spaghetti Monster if we like. But we cannot reject the God of Classical Theism without committing ourselves to a fully developed and properly defended philosophy of Materialism, any more than we can reject belief in light without accepting belief in darkness—which is of course, precisely what every Atheist philosopher of any repute in history has labored to produce. David Hume, Friedrich Nietzsche, Bertrand Russell, Antony Flew… these and many other luminaries devoted their lives to producing materialistic philosophies of nature, mind, and ethics based on some, or all of the above counter-axioms, and published countless influential works in the process (Hume, 2000; 2017; Nietzsche, 2000; Russell, 1967; 2017; Flew, 2005 to name a few).

According to Harris and my friend, all of that was a waste of time—what these and countless other luminaries should’ve been doing, was belittling televangelists and suicide bombers on social media and in TED talks to like-minded audiences. They, of course, knew better. Those who insist that there’s no evidence for “a god or gods” are merely demonstrating that they don’t even understand the question, much less have a properly thought out answer for it.5

 

A reporter once presented the late Samuel Shenton, then president of the Flat Earth Society, with a photograph of earth taken by the Apollo 13 astronauts from roughly 150,000 miles distance. Shenton stared long and hard at it, after which he began to nod. “Yes,” he finally said… “It is easy to see how the untrained eye could be fooled by that picture!” Well-trained eyes are becoming an increasingly important part of the modern intellectual landscape… particularly in secular communities that wear their claims to “reason” and objectivity like golden tiaras. But as I said in my last post, if our only tool is a hammer then sooner or later everything will look like a nail. Though some would deny it (sincerely, I believe), to many in these communities, science is no longer a discipline. It has become a religion in its own right—Scientism, the sacred Oracle whose mighty outstretched hand no question of earth, sky, heart, or soul can elude. Its practitioners are no longer experts, but authorities—high priests of the goddess Reason, whose metaphysical pronouncements are every bit as authoritative as the theistic fundamentalist dogmas they, often rightly, deride.

Nowhere is this more true than with physics—a discipline that not only knocks on the door of many metaphysical questions, but immerses itself in counterintuitive mysteries that at times seem almost magical, and higher mathematics that to the guy on the street are every bit as arcane as ancient hieroglyphics… so much so that a term has even been coined for it: physics envy. And human nature being what it is, once a scientist has been elevated from mere expertise to the august status of High Priest, he/she becomes an authority not only in their own field, but in beer brewing, Elizabethan poetry, personal lubricants, or any other topic for which it’s their whim to have an opinion. Anymore, hardly a week goes by that I don’t see yet another news story extolling Stephen Hawking’s latest complaints and/or warnings about society, international politics, or the impacts of technology on the future of humanity—as though expertise in quantum cosmology qualifies him to speak to any of those topics. [That isn’t Hawking’s fault of course. Scientists rarely ask for the deification so glibly bestowed on them by a credulous public.]

Unfortunately, there’s one big problem with all this… Like it or not, science is a discipline, not an Oracle. A powerful discipline to be sure, and one that has rolled back the mysteries of the universe like no other, but a discipline nonetheless, and for damn sure, no more either. And like all other disciplines, it is, and always will be, but one tool among many. As such, it lends itself to many but not all questions, and the experts who wield it are fallen mortals every bit as subject to their own hopes, fears, and human limitations as we are. It’s the height of naivete and outright hubris to pretend that we can cleanse it of our own limitations and treat it like a magic wand that can answer every question, meet every moral, spiritual, and existential need, and endow our existence with purpose… and we pay a steep price when we do. The philosopher Alfred North Whitehead once said,

“Scientists, animated by the purpose of proving they are purposeless, constitute an interesting subject for study.”

True that.

 

Footnotes

1)   I’m not knowledgeable enough about Hinduism to speak with any authority about it, but its concept of Brahman as the Absolute appears to bear some similarity to the God of the Abrahamic traditions. If so, then including it in this list would raise the tally of humanity that embraces some version of the God of Classical Theism to nearly 70%.

2)   There is one version of the cosmological argument that does presume that the universe had a beginning—the Kalam cosmological argument whose most notable proponent is William Lane Craig. However, it isn’t based on time-ordered causality either. The Kalam argument differs from the traditional one in that it contains two additional premises: Whatever begins to exist has a cause; and that this cause must be transcendent because (per Parmenides) the universe cannot efficiently cause itself. But like the traditional cosmological argument, it takes this cause to be essentially-ordered as well.

3)  Conservation of energy is suspect even for a flat universe. In this case, the global energy of the universe can be derived from the Poisson equation, which has no solution for an unbounded fluid. There is one, and only one case in which the universe can be said to have a well-defined global energy, and that is if it’s closed, in which case, a global definition of energy/momentum flux (gravitationally equivalent to Gauss’ Law) would require it to be zero.

4)  Interestingly, some physicists and philosophers are now beginning to question this, and their reasons are rather surprising. In recent years, multiverse models based on eternal inflation and the so-called string landscape have in the eyes of many physicists, become “the best game in town” for a “theory of everything” that could potentially resolve many issues in physics and cosmology. The inflationary framework accounts beautifully for a few cosmological conundrums that would otherwise be inexplicable (e.g. – the “flatness" problem, and the uniformity of the cosmic microwave background). But in the absence of a viable candidate for the inflaton (as of this writing), the scalar potential/s in inflationary models are flexible enough that for the time being at least, validating the framework has largely proven to be a whack-a-mole exercise. For every model that’s been observationally ruled out, more have sprung up. Likewise, while string theory has led to much progress in many areas, it has also proven excessively flexible—so much so that since its inception more than 40 years ago, it has yet to make a single testable prediction. Furthermore, the scale on which it’s real nuts and bolts are expected to reveal themselves requires testing at energies that will never be accessible to us (Woit, 2007). For all intents and purposes, this renders string landscape multiverse models virtually untestable… even in principle. However, in spite of these problems, they offer two really big carrots that in addition to their other strengths have proven irresistible to many physicists: a) In conjunction with anthropic arguments, they currently offer the only workable explanations of fine tuning that are based solely on physics; and b) Though vulnerable to some formidable arguments that the universe had a beginning, eternal inflation does offer at least some hope for avoiding a creation event. Technically, “eternal” inflation is a reference to future-eternal inflation and thus a bit of a misnomer. A past-eternal universe would run afoul of the BGV theorem; there are a few ways to get around it, although the best of them are contrived to say the least.

The bottom line is that as of this writing, the string landscape/eternal inflation multiverse offers the only path forward for cosmology that doesn’t smack of a Creator. Given the theistic alternatives, it’s little wonder that many Atheist physicists (most notably Sean Carroll) are willing to accept these limitations and argue that it’s time to dispense with testable predictions in science. If a theory is “elegant” (in their view) and at least fits observation, it is de-facto true. Likewise, it also comes as no surprise that many of the strongest opponents of this movement (known as Post-Empiricism) are Christians like George Ellis (Ellis & Silk, 2014).

Ironically, the shoe is now on the other foot. Atheists who for so long have (often rightly) accused religious believers of clinging to comfortable dogmas without evidence, are now the ones insisting that science should be divorced from it. When their backs are against the wall (and to their credit IMHO), they prove to be every bit as mortal as people of faith. And like us, they cherish their worldviews enough that they’ll occasionally struggle for their preservation even to a fault.

5)   Antony Flew is a particularly telling case in point. Often referred to as the Father of 20th Century Atheism, he was arguably the most important Atheist philosopher of his age. His seminal work God and Philosophy (2005), which was originally published in 1966, almost single-handedly shaped the direction of Atheist thought and scholarship during his lifetime. Shortly before his death in 2010, he shocked the secular world when he set aside his life’s work and said that based on reason and evidence, he could no longer deny the existence of God (Flew & Varghese, 2008). Flew didn’t conclude with a God who is personal, as in the Bible and Quran, nor did he embrace any major religion. But his God did bear a striking similarity to the God of Classical Theism, and he gave a particularly deferential hat-tip to… Christianity.

Needless to say, this dealt New Atheists a narcissistic injury which they still haven’t recovered from to this day. The reaction was immediate, and what one would expect. Despite his life’s work, Flew was promptly branded an apostate to the True Faith and excommunicated. Dawkins (2008) fumed about his “tergiversation” (as though using the biggest and most impressive word he could find in a crossword puzzle would somehow convert bullshit into a valid argument). Others resorted to smear campaigns (up to and including accusing him of senility), and intellectual cross-burnings that would make even the flock of Westboro Baptist Church blush. The one thing that was not, and to this day has not been produced, is a properly researched and soundly defended critique of his stance.

Perhaps New Atheists are as offended by religion as they are because they have more in common with blindly dogmatic religious fundamentalists than they’re prepared to admit. Few people evoke as much hate as those who hold a mirror up to us that we don’t want to face.

 

References

Armstrong, K. (2015). Fields of blood: Religion and the history of violence. Anchor; Reprint edition (September 15, 2015). ISBN-10: 0307946967; ISBN-13: 978-0307946966. Available online at www.amazon.com/Fields-Blood-Religion-History-Violence/dp/0307946967/ref=tmm_pap_swatch_0?_encoding=UTF8&qid=1499969508&sr=8-1. Accessed July 13, 2017.

Carroll, S. 2010. Energy Is Not Conserved. Discover, Feb, 22, 2010. Online at http://blogs.discovermagazine.com/cosmicvariance/2010/02/22/energy-is-not-conserved/#.WkLCkkqnFaQ. Accessed Dec. 26, 2017.

Carroll, S. (2017). The Big Picture: On the Origins of Life, Meaning, and the Universe Itself. Dutton; Reprint edition (May 16, 2017). Chap. 3. ISBN-10: 1101984252; ISBN-13: 978-1101984253. Available online at www.amazon.com/Big-Picture-Origins-Meaning-Universe/dp/1101984252/ref=mt_paperback?_encoding=UTF8&me=. Accessed Dec. 27, 2017.

Carroll S. & W. L. Craig. (2014). “God and Cosmology: The Existence of God in Light of Contemporary Cosmology”. New Orleans Baptist Theological Seminary, New Orleans, LA – March 2014. Transcript available at www.reasonablefaith.org/god-and-cosmology-the-existence-of-god-in-light-of-contemporary-cosmology. Accessed July 14, 2017.

Copan, P., & Craig, W. L. (2004). Creation out of nothing: A biblical, philosophical, and scientific exploration. Baker Academic (June 1, 2004). ISBN-10: 0801027330; ISBN-13: 978-0801027338. Available online at www.amazon.com/Creation-out-Nothing-Philosophical-Exploration/dp/0801027330/ref=sr_1_1?ie=UTF8&qid=1500324234&sr=8-1&keywords=Creation+out+of+nothing. Accessed July 17, 2017.

Dawkins, R. (1996). River out of Eden: A Darwinian view of life. Basic Books; Reprint edition. ISBN-10: 0465069908; ISBN-13: 978-0465069903. Available online at www.amazon.com/River-Out-Eden-Darwinian-Science/dp/0465069908/ref=sr_1_1?s=books&ie=UTF8&qid=1499814281&sr=1-1&keywords=river+out+of+eden. Accessed July 11, 2017.

Davies, B. (2004). An Introduction to the Philosophy of Religion. Oxford University Press; 3 edition (January 8, 2004). ISBN-10: 0199263477; ISBN-13: 978-0199263479. Available online at www.amazon.com/Introduction-Philosophy-Religion-Brian-Davies/dp/0199263477/ref=sr_1_3?s=books&ie=UTF8&qid=1499974934&sr=1-3&keywords=brian+davies. Accessed July 13, 2017.

Dawkins, R. (2008). The God Delusion. Mariner Books; Reprint edition, ISBN-10: 0618918248; ISBN-13: 978-0618918249. Available online at www.amazon.com/God-Delusion-Richard-Dawkins/dp/0618918248/ref=sr_1_1_title_1_pap?s=books&ie=UTF8&qid=1408044395&sr=1-1&keywords=god+delusion. Accessed July 11, 2017.

Ellis, G., & Silk, J. (2014). Scientific method: Defend the integrity of physics. Nature, 516(7531). Available online at www.nature.com/news/scientific-method-defend-the-integrity-of-physics-1.16535. Accessed July 11, 2017.

Feser, E. (2010). The last superstition: A refutation of the new atheism. St. Augustines Press; 1St Edition edition (December 10, 2010). ISBN-10: 1587314525; ISBN-13: 978-1587314520. Available online at www.amazon.com/Last-Superstition-Refutation-New-Atheism/dp/1587314525/ref=tmm_pap_swatch_0?_encoding=UTF8&qid=1499974707&sr=8-1. Accessed July 13, 2017.

Feser, E. (2014). Scholastic Metaphysics. Editions Scholasticae. ISBN-10: 3868385444; ISBN-13: 978-3868385441. Available online at www.amazon.com/Scholastic-Metaphysics-Contemporary-Introduction-Scholasticae/dp/3868385444/ref=sr_1_3?ie=UTF8&qid=1464406953&sr=8-3&keywords=feser. Accessed July 13, 2017.

Feser, E. (2015). Neo-scholastic Essays. St. Augustines Press; 1 edition (June 30, 2015). ISBN-10: 1587315580; ISBN-13: 978-1587315589 Available online at www.amazon.com/Neo-Scholastic-Essays-Edward-Feser/dp/1587315580/ref=pd_sim_14_3?ie=UTF8&dpID=51vOUR5k8eL&dpSrc=sims&preST=_AC_UL320_SR214%2C320_&psc=1&refRID=MP3S70WMRDF7N9VQNPMA. Accessed July 15, 2017.

Flew, A. (2005). God and philosophy. Prometheus Books (April 8, 2005). ISBN-10: 1591023300; ISBN-13: 978-1591023302. Available online at www.amazon.com/God-Philosophy-Antony-Flew/dp/1591023300/ref=mt_paperback?_encoding=UTF8&me=. Accessed July 21, 2017.

Flew, A., & Varghese, R. A. (2008). There is a God. HarperOne; unknown edition (November 4, 2008). ISBN-10: 0061335304; ISBN-13: 978-0061335303. Available online at www.amazon.com/There-God-Notorious-Atheist-Changed/dp/0061335304/ref=tmm_pap_swatch_0?_encoding=UTF8&qid=1500661815&sr=1-1. Accessed July 21, 2017.

Harris, S. (2008). Letter to a Christian Nation. Vintage; Reprint edition. ISBN-10: 0307278778; ISBN-13: 978-0307278777. Available online at www.amazon.com/Letter-Christian-Nation-Sam-Harris/dp/0307278778/ref=sr_1_6_title_1_pap?s=books&ie=UTF8&qid=1408131913&sr=1-6&keywords=sam+harris. Accessed July 11, 2017.

Hume, D. (2017). An enquiry concerning human understanding. CreateSpace Independent Publishing Platform (July 1, 2017). ISBN-10: 1461180198; ISBN-13: 978-1461180197. Available online at www.amazon.com/Enquiry-Concerning-Human-Understanding/dp/1461180198/ref=tmm_pap_swatch_0?_encoding=UTF8&qid=1500660885&sr=8-3. Accessed July 21, 2017.

Hume, D. (2000). A treatise of human nature. Oxford University Press; New Ed edition (February 24, 2000). ISBN-10: 0198751729; ISBN-13: 978-0198751724. Available online at www.amazon.com/Treatise-Human-Nature-Oxford-Philosophical/dp/0198751729/ref=sr_1_13?ie=UTF8&qid=1500660885&sr=8-13&keywords=david+hume. Accessed July 21, 2017.

Martin, C. F. (1997). Thomas Aquinas God and Explanations. Edinburgh University Press; 1 edition (June 30, 1997). ISBN-10: 0748609016; ISBN-13: 978-0748609017. Available online at www.amazon.com/Thomas-Aquinas-Explanations-Christopher-Martin/dp/0748609016/ref=sr_1_1?ie=UTF8&qid=1500142014&sr=8-1&keywords=Thomas+Aquinas+God+and+Explanations. Accessed July 15, 2017.

Misner, C. W., Thorne, K. S., & Wheeler, J. A. (MTW). 2017. Gravitation. Princeton University Press. ISBN-10: 0691177791; ISBN-13: 978-0691177793. Chap. 20.2. Online at www.amazon.com/Gravitation-Charles-W-Misner/dp/0691177791/ref=sr_1_1?ie=UTF8&qid=1514324449&sr=8-1&keywords=gravitation. Accessed Dec. 26, 2017.

Moreland, J. P., & Craig, W. L. (2003). Philosophical foundations for a Christian worldview. IVP Academic; unknown edition (April 28, 2003). ISBN-10: 0830826947; ISBN-13: 978-0830826940. Available online at www.amazon.com/Philosophical-Foundations-Christian-Worldview-Moreland/dp/0830826947/ref=sr_1_1?ie=UTF8&qid=1460753368&sr=8-1&keywords=Philosophical+Foundations+for+a+Christian+Worldview. Accessed July 17, 2017.

Nietzsche, F. (2000). Basic Writings of Nietzsche. Modern Library; Modern Library edition (November 28, 2000). ISBN-10: 0679783393; ISBN-13: 978-0679783398. Available online at www.amazon.com/Writings-Nietzsche-Modern-Library-Classics/dp/0679783393/ref=tmm_pap_swatch_0?_encoding=UTF8&qid=1500661604&sr=1-7. Accessed July 21, 2017.

Oderberg, D. S. (2008). Real essentialism. Routledge; 1 edition (January 30, 2008). ISBN-10: 041587212X; ISBN-13: 978-0415872126. Available online at www.amazon.com/Essentialism-Routledge-Studies-Contemporary-Philosophy/dp/041587212X/ref=sr_1_2?ie=UTF8&qid=1500142092&sr=8-2&keywords=Thomas+Aquinas+God+and+Explanations. Accessed July 15, 2017.

Russell, B. (1967). History of Western Philosophy. Simon & Schuster/Touchstone (October 30, 1967). ISBN-10: 0671201581; ISBN-13: 978-0671201586. Available online at www.amazon.com/History-Western-Philosophy-Bertrand-Russell/dp/0671201581/ref=mt_paperback?_encoding=UTF8&me=. Accessed July 21, 2017.

Russell, B. (2017). The problems of philosophy. CreateSpace Independent Publishing Platform (April 21, 2017). ISBN-10: 1545507635; ISBN-13: 978-1545507636. Available online at www.amazon.com/Problems-Philosophy-Bertrand-Russell/dp/1545507635/ref=mt_paperback?_encoding=UTF8&me=. Accessed July 21, 2017.

Wikipedia. (2016). Greek primordial deities. Available online at en.wikipedia.org/wiki/Greek_primordial_deities. Accessed July 17, 2017.

Woit, P. (2007). Not even wrong: The failure of string theory and the search for unity in physical law. Basic Books; Reprint edition (September 4, 2007). ISBN-10: 0465092764; ISBN-13: 978-0465092765. Available online at www.amazon.com/Not-Even-Wrong-Failure-Physical/dp/0465092764/ref=mt_paperback?_encoding=UTF8&me=. Accessed July 16, 2017.

 

discipline 

About Scott Church

I am a landscape photographer and I.T. professional in the greater Seattle area. I graduated from the University of Washington with a Bachelor's in Mechanical Engineering and a Masters in Applied Physics, and in a former life, I was an aerospace engineer. When I'm not writing or at work I can be found plying the waters of the Pacific Northwest for salmon, trout, and steelhead, or bushwacking with my camera gear.
This entry was posted in Metaphysics, Theology. Bookmark the permalink.

95 Responses to Will the real god God please stand up?

  1. Spencer Andreasen says:

    You had me as 'bucolic'.

    I'm interested in your perspectives on Clinate Change after following one the links in this article.

    Most of the physics flew over my head but I'm more or less just glad knowing that there are smart Christians keeping the smart Atheists honest, so well done (and to Aron).

  2. Scott Church says:

    Thanks so much for the kind words Spencer! For my perspectives on climate change, you could do worse than perusing the stuff I've posted on my own website, (which is probably the link you followed). The first link on that page is my latest writing on the subject, an essay about some of the common threads running through most climate denial arguments. There's also a link on the right nav bar to a general climate science section.

  3. Andrew says:

    I don't think its quite fair to categorize people who argue something can come from nothing as "metaphysically illiterate physicists", Quentin Smith, J.L. Mackie, Graham Oppy, Daniel Dennett, have all at least entertained the idea. Their arguments might be fatally flawed but they're not metaphysically illiterate – maybe I am – but I can't see any contradiction or incoherence in the claim that something can come into being from _literally_ nothing.

  4. Nathan says:

    Hello Scot and/or Aron,
    I am curious how B-theorists deal with ideas of God creating and causing things, given that most B-theorists seem to affirm timelessness and changelessness. In a prior post of Aron's called "Time as a fourth dimension" - http://www.wall.org/~aron/blog/the-geometry-of-spacetime-i-distance/ One commentator asked Dr. Wall this, "..we can definitely and surely say there is no possible notion of causality, time and change, indeed 4D Euclidean pure space is necessarily and absolutely A-causal ( Causality-free) ,timeless and changeless", to which Dr. Wall responded with, "Yes Kaveh, that's more or less right. Although you could still describe a field as "changing" in the sense that its value depends on one of the spatial coordinates, say x. And, given enough "initial data" about the fields at x=0, you can usually use the field equations to determine the values of the fields at any other value of x.". So, my question is simple, if the space-time of the universe (and presumably the events within it) is without cause and is timeless and changeless (and even the word "necessary"), then what room for God? It certainly seems to undermine many of the arguments I have seen on this blog and in other work of Christian thinkers. Why can't the universe just be timeless and changeless without God?

    [I rescued this comment from the spam filter--AW]

  5. Nathan says:

    Hello Scott or Aron,
    I have tried to post this comment a few times, but I seem to fail each time for some reason. My question is rather simple, if the B-theory of time implies that the universe is timeless and changeless, why can't it just exist without God? Why would we need a God to do anything if the underlying spacetime just exists timeless and changelessly? Can't the atheist just say that the B-theory allows the universe to exist a-causally (without a cause)?

  6. Nathan says:

    Here is a link to Aron saying something similar to the universe being timeless, unchanging and a-causal - http://www.wall.org/~aron/blog/the-geometry-of-spacetime-i-distance/

  7. Scott Church says:

    Hello Andrew and Nathan. My apologies for the delay in response. It's been a busy weekend! I'll respond to your comments soon as I have two minutes to rub together. Thanks in advance for your patience! Best. :-)

  8. Robert Childress says:

    Good read!

  9. Scott Church says:

    @Andrew

    I'm aware of the views of Oppy, Smith, and the others you mention regarding creation from nothing, and I do agree--they certainly aren't metaphysically illiterate. However, they weren't who I was referring to. The people who first put the idea of the universe creating itself from nothing (what some have called the "free lunch" universe) on the map and popularized it were physicists. Though these men are very accomplished in their fields and I don't want to detract from that, to a man, all of them are metaphysically illiterate (sorry... but I see no valid reason not to name this for what it is). In fact, the one who did more than anyone else to popularize the idea over the last decade or so, Lawrence Krauss, takes metaphysical illiteracy to a whole new and terrifying level. That's not just my opinion either. He's been called out repeatedly by other Atheists as well. And although I hate to say it, much as I respect and look up to Hawking, he's developed a reputation for his lack of metaphysical understanding too.

    What sets these men apart as metaphysically illiterate isn't just that they think things can create themselves from complete non-being. It's that they actually think "nothing" can be brought within the purvey of physics simply by redefining the word so that it magically becomes a something with actually properties. I've stood by and watched in utter amazement as Krauss went back and forth equivocating between something and "nothing" multiple times in single sentences. No one with even a minimal grasp of the English language and Logic 101 (much less metaphysics) would ever be fool enough to attempt such sleight of hand and think they could get away with it.

    As for the claim that something can "come into being from _literally_ nothing", I would agree that technically, there's no inherent contradiction or incoherence in it. But I think we need to call that what it is--magic. I can imagine how someone could be metaphysically literate and still believe in magic. For that matter, I can see how he/she could believe in the Tooth Fairy as well. But I find it interesting, and revealing, that the moment science girds up their loins and takes them where they do not want to go (John 21:18), otherwise brilliant Atheists who rightly pride themselves in their commitment to science, will openly refuse to educate themselves in fields beyond their expertise... and take refuge in magic rather than face the logical possibility of a transcendent Creator.

  10. Scott Church says:

    @Nathan,

    "Can't the atheist just say that the B-theory allows the universe to exist a-causally (without a cause)...?" Technically, yes. Essentially, that amounts to embracing axioms 8) through 10). But doing so comes at a rather steep price.

    For starters, it doesn't address the contingency of the universe--that is, the fact that its essence, or actual nature, is unrelated to its existence. Of all ways that it could've been, why is it like this rather than any of an infinite number of other possible ways, the overwhelming majority of which are chaotic? Whence what Wigner referred to as "the unreasonable effectiveness of mathematics"?

    Then there's the fact that 8) through 10) invariably compel one to embrace some combination of 11) through 16) too, which in turn, necessarily requires one to compound the list of "just so" claims. Furthermore, by definition, space-time doesn't exist "timelessly", and although they may give lip service to the idea of non-causality, in the real world, few physicists live as though the world isn't causal. The very term law of nature presumes at least some concept of causality. Typically, most physicists would consider two events causally related if they're connected by a timelike world line, related to each other by a known law/s of physics (and requisite boundary conditions, if applicable), and a time asymmetry exists between them per the Second Law. To dispense with that at the moment of creation is to commit a taxicab fallacy and/or arbitrarily dismiss outright everything but the notion of accidentally-ordered efficient causality, thereby making a metaphysic out of science while claiming to denounce metaphysics.

    IMHO, that's quite a mess. It leaves the Atheist in the unenviable position of having to draw numerous "just so" face cards from the deck, and then attempt to reconcile multiple internal inconsistencies in a convincing manner. They certainly can do that if they wish. But the God of Classical Theism as defined by a) through g) already accounts for all of this without any such antics, and is directly derivable from the damn reasonable list of seven axioms we began with. Theists can just play the hand reality dealt them, and call the Atheist's bluff. :-)

    @Robert, thank you! :-)

  11. Nathan says:

    Thank you for the reply and I apologize in advance for the noob-question, but what are these axioms you are talking about and where is a list that I can read? You say, "Technically, yes. Essentially, that amounts to embracing axioms 8) through 10). But doing so comes at a rather steep price. " What is axiom 8 and why does it lead to "16"? Sorry for the foolish question, but I am new to this. Is there a page or book that I can read which will list and explain these axioms? Anyways, thank you for your response.
    P.S. I am not an atheist and I do believe in the reality of change, I just wanted to know what a B-theory theology would look like!

  12. TY says:

    Scott,
    I have always been bothered by the “non-classical” argument that God is contingent, a view shared by serious thinkers such as Swinburne and Don Page, though they make up a minority in the theism camp. The non-contingency puts the God of classical theism on the same level as Zeus, the golden calf, Apollo, etc. in that they could or could not have existed.

    St Paul’s famous Areopagus sermon nails down the argument for God being necessary:
    “The God who made the world and everything in it is the Lord of heaven and earth and does not live in temples built by human hands. And he is not served by human hands, as if he needed anything. Rather, he himself gives everyone life and breath and everything else.

    My question is: Is a contingency God incompatible with theism and the fundamental attributes of the God in classical theology?

    Thanks

  13. Scott Church says:

    Hi @Nathan,

    No worries... your question is most certainly not a "noob" one! :-) Perhaps I could've been clearer... the axioms I'm referring to are in this post. The first 7 (beginning with paragraph 6, by my count) lead to the God of Classical Theism, which I summarize in points a) through g). Later on, in the section titled "Atheism is not a null hypothesis" I give 8) through 16) as their counter-axioms, some combination of which Atheists are compelled to embrace if they want to be consistent in their rejection of God. The source material cited in my references go into considerable length on the formal details of all this, particularly the works of Feser and Davies (2004).

    Best. :-)

  14. Scott Church says:

    Hello @TY,

    The "non-classical" argument you refer to is the Theistic Personalism I wrote about above. Technically, it doesn't really put God on the same level as demigods like Zeus or Apollo because the latter are still bound by space and time. Most theistic personalists would argue that unlike them, God is greater than, and beyond His own creation, as He must be. Where they part company with mainline Classical Theology is their belief that He cannot be meaningfully personal unless He isn't simple or changeless, and per the latter, in some sense has His own experience of time. As you noted, this would render Him contingent in that whatever can change in this way could conceivably change in that one as well, and we'd be left having to account for the difference. That, at least, He'd have in common with Zeus, etc.

    Is this contingent God compatible with Classical Theism? Depends on who you ask. Some, such as Feser (2014; 2015) and Davies (2004) would say no, and inasmuch as that tradition is built predominately around the first seven axioms I listed and arguments based on them, they're right. But on the other hand, a case can be made for a contingent personalist God that persons like us could more easily relate to.

    As I noted above, the arguments for God's simplicity and changelessness are formidable. But I believe the biggest problem with arguments for God's contingency and changeability, is that they all but undermine the case for His existence. Atheists have repeatedly, and rightly, pointed out that a contingent God is no better a candidate for the ground of all being than the universe, and we have every right to ask who/what created Him as well. Stopping with the theistic personalist's God is no less a taxicab fallacy than stopping with the universe... and clearly, the latter does exist.

    Believers will counter that Scripture, by which God has made Himself known to us, is the deciding factor (along with whatever historical support can be gleaned from it, and external sources). Technically I would agree, but this argument is only going to be compelling to those of us who're already believers for other reasons. And for most of us, those other reasons are largely (if not completely) based on our own hearts, prayers, and existential life experience, and these are subjective enough that we'd be hard-pressed to even protect them from being circular, much less ground them solidly before the secular world.

    Ergo, IMHO, and with all due respect for its luminary proponents, Theistic Personalism has no teeth.

    Best. :-)

  15. TY says:

    Scott. I agree with the incompatibility answer. That has been my log-held view. My only problem is I have seen any arguments for Contingency so I can determine where the proponents are coming from, or whether they just want to be eccentric.
    Thanks.

  16. Scott Church says:

    Hello @TY.

    If you want to expose yourself to the strongest arguments for a contingent, personalist God, its most noteworthy proponents (at least, that I'm aware of) are William Lane Craig, Richard Swineburne, and Alvin Plantinga. An introduction to Craig's arguments can be found in Moreland & Craig (2003) in my references, or for a quicker read, here. Of the works of his that I've seen, Swineburne's views are probably best explicated in his book The Existence of God. Ed Feser has responded to Craig's arguments for theistic personalism here, including a recap of many of Craig's thoughts, and answered other theistic personalists in other blog posts as well (including this one).

    Somewhere in all of that and the sources cited therein, you should be able to find the best arguments for a contingent God that have been put on the table to date.

    Best. :-)

  17. Andrew says:

    Thanks for your response, Scott. I can't help but notice your other comments, I would add Robin Collins in as someone who give one of the best defense's for theism: http://citeseerx.ist.psu.edu/viewdoc/download?doi=10.1.1.696.63&rep=rep1&type=pdf His formulation of the fine tuning argument is quite strong.

  18. Angela says:

    I am enjoying your posts here. :) I have read Feser's responses (the links you refer to) to Craig on simplicity a few times and have wrapped my head around some of it but it keeps getting fuzzy in regards to change, pure actuality and creation of the universe. Two commenters (the last, most recent ones) on the Feser response to Craig from 2009 posit my question well. (There are only 50 comments on this post as opposed to the 250 on the other one linked to.) I believe it's probably a lack of philosophical understanding on my part but I would love some clarification. Thank you.

  19. Scott Church says:

    Hello @Angela.

    Thanks for the kind words. I'm having trouble locating the comments at Feser's blog you're referring to. As of this writing, of the two posts of his I linked above, neither has only 50 comments and the most recent ones I see don't ask him any questions. If you could find the two that you felt best captured your question and paste them here, I'll do my best to address them.

    Best. :-)

  20. Calhoun says:

    Hi Scott.
    Have you engaged with work of Philosopher Ryan Mullins ? He is the best critic of classical theism I have read. If not, you should check out his book "End of Timeless God" most of its material can be found in his dissertation "In search of Timeless God" which can be found online and also Here is his case against Divine simplicity.

    And he has also argued Here that Four-dimensionalism or Eternalism (I think that is what users here mean by B-theory) is Rationally Incompatible with Christian Theism.

    I would be very interested in what classical theists think about it, I haven't seen any classical theist deal with his work before. I myself like classical theism a lot but I think Mullins present convincing arguments against it.

    Thanks..

  21. dad says:

    Very interesting and comprehensive reading.
    Dad

  22. Tom Rudelius says:

    Hey Scott, nice post--very well-written and well-researched. I can't help but wondering how you respond in real-life situations when someone asks you how you can believe in '"a god or gods” in an age of “science” and “reason”'? I'm pretty sure this post is too long for a face-to-face conversation :)

  23. Tom Rudelius says:

    Sorry, another couple of questions regarding theistic personalism: First, Dr. Craig has advocated the contingency argument, which only works if God is a necessary being. How does one square this with his view of theistic personalism, which you claim requires a contingent God? Second, as someone who rejects theistic personalism, how do you bridge the gap between the necessary, simple, omnipotent, eternal being of classical theism and the "God" of scripture, who possesses a personality and generally seems to interact as a character in the play (and not merely as the playwright)?

  24. TY says:

    Scott,
    What do you say to the atheist or the skeptic that the theistic argument of God being a “necessary being” is a trick to assert that God is uncaused. If God is caused or contingent, we run into an infinite regress. So, to get around this "non-explanation", theists invoke the notion of a "first cause", which must be uncaused. This first cause is what theists mean by God. But in the end, it's no more than question begging.

    Thanks.

  25. Angela says:

    My apologies. It must have been a rabbit trail/link from a link kinda thing.
    Here is Feser's post I was referring to:
    http://edwardfeser.blogspot.com/2009/11/william-lane-craig-on-divine-simplicity.html
    (Sorry for lack of proper formatting, I'm out of my depth on this site in many regards.)
    The commenters (the third and fourth from the last as of today's date) ask how God is changeless/lacking potential given that He created the universe, for example. One comments that this challenge is "the killer" against divine simplicity. That sounds like a strong assertion, but I'm unable to articulate to the contrary.
    Thank you again!

  26. Mactoul says:

    Scott,
    The universe may be defined as the totality of consistently interacting things. It is not a thing itself. Hence I am unable to understand what precisely is meant by your statement:

    1. The universe is contingent. Its essential nature, or form (and that of everything in it) is separate from its existence. [e.g. - We can meaningfully conceptualize horses and unicorns without regard to whether there are any.]

    It is the things in the universe that are contingent. They have natures or forms. But can the universe be said to have an "essential nature or form"? How would you go on defining the form of the universe?

  27. Scott Church says:

    @Calhoun,

    I'm not familiar with Ryan Mullins so I can't speak (yet) to how his arguments differ from those of other theistic personalists. But I'll check out the links you provided and address them when I get the chance. Thanks!

    @Tom,

    Thanks so much for the kind words. And how right you are... responding to such questions succinctly in real-life situations is challenging to say the least! Especially for folks like me, who have what a former colleague used to refer to as "book where a pamphlet is required" disease. Have you ever come across a student or non-physicist lay person who wanted to know why they should believe all this crazy relativity and curved space-time stuff, instead of the first-semester physics they learned in high school... and wanted you to thoroughly convince them before either of you finished your coffees? It reminds me of a meme I saw years back... A 1 cm square, under which it said, "State the nature of your complaint in the space provided. Write legibly, and give full details". Perhaps there are people who could do that... but I certainly am not one of them. I just do the best I can and hope they don't fall asleep on me and drive off the road. :-)

    Moving right along, let me see if I can't answer your questions and @TY's together...

    First, how do we square Craig's contingency argument with his view of theistic personalism? We don't... and therein lies what IMHO is theistic personalism's Achilles heel. There is no way to ascribe the complex subtleties of a personality like our time-bound ones to God without raising the question of why He's this sort of "guy" and not like someone else. By definition, that question is inherently contingent, just like any other about the universe. There's no longer any way to avoid the infinite regress @TY referred to without resorting to a taxicab fallacy, and if the cab has already been called and we're on the curb with our bags, why not Just get off at the stop we already know? Why run the meter up for another contingent stop we can't see?

    The key to all this is: a) Essentially vs. accidentally ordered causality; and b) Active vs. passive potencies. Consider the ball/stick example I gave. The ball has the capacity for rolling motion and the stick for pushing it. But these are passive potencies in that neither can be realized on their own. A ball can't roll itself--it needs the stick to push it. Likewise, the stick can't push the ball by itself--it needs something or someone to wield it for that purpose. We, on the other hand, have the power to pick up the stick and wield it to push the ball--an active potency. We also have passive potencies, such as the capacity for warmth, but only if something else (e.g. a campfire) provides the heat.

    By definition, passive potencies imply contingent dependence. The moment we ascribe properties to something (in the usual sense of that term), we're left with the question of what other cause/s led to this, rather than that state. And... an infinite regress or causal loop doesn't fix things. Consider the train example I mentioned in axiom 5). Freight cars have the capacities for pulling other cars and being pulled themselves. But none of them can induce motion on their own, individually or collectively. An infinitely long train of freight cars, or one connected end-to-end in a loop, cannot move itself--there must be at least one engine possessing active motive power.

    The only way to cleanse such an essentially ordered (i.e. simultaneous) causal chain of contingency-driven taxicab fallacies is to terminate it in something that is purely actual grounded being--uncaused by anything else, unchanging and unmotivated by anything outside of itself, and dependent on nothing else for its essence and empowerment... Pure Act.

    @TY, this is why a necessary being is in fact necessary, not just a "trick". Notice how many of my materialistic counter-axioms 8) through 16) involve blind assertions--"it just is like that..." No such claims appear in axioms 1) through 7), all of which are based on our observed experience of the existence, and behavior of the universe around us.

    It's also why I suspect you'll be hard-pressed to find an Atheist who will tolerate any discussion of the traditional cosmological argument--as it was developed by Aristotle, Aquinas, and the Scholastics. If your experience is like mine, in every case they'll try to shift the argument to one based on an accidentally ordered temporal first cause, in which case contingency is unavoidable even in principle (this is why Aquinas famously rejected it). The reason is clear enough. If they can restrict the discussion to this knock-off argument, they can handicap Theists with their own contingency problems, thereby giving themselves a fighting chance. Theists, of course, have no reason to accept such a handicap simply because Atheists need them to so they can keep up... and when they don't, things tend to go south rather quickly.

    Case in point, the last time I presented the cosmological argument to an Atheist, it was to a friend on Facebook. He'd asked me for "[my] best argument for the existence of god" (note the lower-case "g"), after which, he said, "[he'd] tell [me] what's wrong with it". [Note the hubris in presuming my choice to be flawed before he even knew what it was]. I did so, carefully pointing out the difference between essentially ordered and accidentally ordered causal chains, and citing all the scholarly source material he'd need to understand it. Without missing a beat, he responded to the knock-off temporal cause argument instead. When I asked why, he said the knock-off was "how most people understand it", to which I replied that he'd asked for my best argument for God's existence--not his, or anyone else's favorite bad one. There were a few more failed attempts to get me to take the bait. [Sorry... been there, done that, ain't fallin' for it ;-) ]. The conversation ended when he flatly refused to discuss anything but the knock-off argument, and wouldn't read any of the source material I'd provided. Every discussion I've ever had with Atheists regarding the cosmological argument has more or less followed this script.

  28. Scott Church says:

    @Tom, to your second question, I believe it answers itself once we realize that God is eternal, and as our physics requires, beyond the space-time He created. To say that is, in essence, to say that He is everywhere, and at every time, imminently present in all of it. Our entire history is known to Him as one eternal "present" which He sustains and knows at every event. By contrast, we are time-bound creatures whose thoughts, choices, and interactions with creation find their expression in the unfolding of our stories in history. We have no direct experience of personhood apart from our personalities, which are inseparable from our historical, time-bound stories. A God who knows and loves us, and is imminent at every point on the world lines of our stories, will hold us in His hands differently at every event. As time-bound creatures, our experience of that will be one of perceived change, just as passengers in a car driving through a static landscape will experience their passing through it as changes in scenery.

    We must also remember that as mortal human beings whose experience of the universe's deepest secrets is limited, our language will be as well, and as such, will necessarily require some metaphor. This isn't just theological hand-waving either--we see it in our physics. As I believe my first series made clear, our ability grasp QM is fundamentally limited as well--to the point that our language cannot fully explicate it. In some contexts, we characterize the Higgs as a particle, in others, as a field. In both cases, it's understood that in reality, the Higgs is something larger than both, and our discussions of it are to some extent necessarily metaphorical. If this is true of our attempts to wrap our minds around the universe, how much more will it be so for biblical writers putting words to convey the one who created it?

    @Angela, I believe this also addresses "the killer" argument against divine simplicity you found in the comments on Feser's post (thanks for the link!). Commenter "JC" asks, "Doesn't creating the world affect God at least in his self-understanding as coming to know (eternally) that He created the world...?" Not that I can see. I see no reason at all why God cannot create the universe any way He pleases from His own active powers, and still be the pure unchanging substance that He is. Furthermore, I believe JC gives away the game with the phrase "coming to know"... "eternally". How exactly does eternal knowledge "come into" being? This strikes me as a muddle, not a "killer" argument. For a more rigorous analysis of the issues involved here, see Feser (2014; 2015).

  29. TY says:

    Scott,
    Thanks for the very thoughtful responses in this post.

    Just a final comment. You said: "From these (particularly the concept of essentially-ordered causality), they concluded that there must exist something that is pure act—the ground of all being and empowered possibility, with no passive potentialities or dependencies (Davies, 2004; Feser, 2010; 2014)." It seems to me that the "must" in the quotation would make the atheist or skeptic shout "non-sequitur", which might be another way of saying that the existence of God cannot be logically or mathematically proven. But the fact that no such proof is possible is not the same as saying "there is no evidence" for God, and that to me is the main take-away from this fine post.

  30. Andrew2 says:

    Hi Scott! Really great post here. Some of your comments on the cosmological argument have reminded me of some concerns I've had with it for a while - I'd be interested to hear your thoughts:

    You move from talk of essentially ordered causal chains, to those chains requiring a termination in a being that is Pure Act, i.e. one with no potencies - a mover that is not itself moved. But I don't understand how this step is supposed to work.

    To illustrate what I mean, take your example of the freight cars - they have no active power, and so there must be at least one locomotive in the chain somewhere that's providing that power. I can grant this chain is essentially ordered, but I don't see how it helps the claim that essentially ordered causal series ultimately terminate in Pure Act. Surely this specific chain terminates with the locomotive? After all, the point of the chain is that the various members that are merely instrumental causes, like the freight cars, are dependent for their motion on something with actual motive power, namely the locomotive. But the locomotive isn't Pure Act - it has its own passive potencies (e.g. it can rust). Perhaps your response here would be that the real head of the chain isn't the locomotive, but whatever it is that's powering it (e.g. burning diesel, or something). The burning diesel might not be the head of the chain, of course - it is itself causally dependent on other things (e.g. a spark to light it), but it's not clear whether that cause forms part of the essentially ordered series, or a different accidentally ordered one (once the spark has lit the diesel, the diesel will keep burning until it either runs out or it's extinguished - it doesn't continue to depend on the spark in order to keep burning). Regardless, the diesel also has its fair share of potentiality (e.g. it is potentially frozen).

    (Brief aside: this brings me to another point - it's never been clear to me why you can't combine essentially and accidentally ordered series. E.g. if you take the ball being moved by the stick, which is being moved by the person, I could grant that's an essentially ordered series, but it's embedded in an accidentally ordered one - the person was caused to exist by their parents, but doesn't continue to depend on them for their existence now they've been born.)

    Anyway, hopefully this illustrates my larger point, which I'll state more generally. I agree that an essentially ordered causal chain must terminate. That's because each member of the chain has whatever power it does only derivatively, and must derive that power from something that actively has it. But I don't see how we can conclude that the head of the chain must be Pure Act, i.e. lacking any potentiality whatsoever. Yes the head of the chain must be actual with respect to whatever potency is being actualised further down the chain, but why can't it have a potential in some unrelated aspect (like how the locomotive is potentially rusted, while still actively moving)?

    Finally, supposing we can show that any essentially ordered series must terminate in a being that is Pure Act, why couldn't there be multiple such beings? There is surely more than one essentially ordered series in the world - why not suppose that one series terminates in one Pure Act being, and another series terminates in a different Pure Act being?

    I'll also stick a quibble in with your presentation of essentially ordered series - the defining feature of an essentially ordered series is not that causation is simultaneous, but that all its members, except the head, are instrumental causes. After all, it's easy to come up examples where the causation is not strictly simultaneous. E.g. in the freight car example, taking relativity into account, each car won't actually simultaneously push/pull its neighbour, but there'll be a small amount of time between each event, as the impulse is 'transmitted'. Alternatively, consider the following scenario: a moon is shining, because it's reflecting light off another moon, which is in turn shining because it's reflecting light off another moon, and so on. Clearly, even if the number of moons were infinite, that wouldn't explain why any of them are shining, because none of them have the power to shine in and of themselves, but only to reflect. So there must be a light source(s), e.g. a sun, that provides light to the whole chain, and if the sun stops shining, so do all the moons. This is an essentially ordered series. But simultaneity isn't what makes it so - indeed, the chain isn't simultaneous, because light travels at a finite speed. It's the fact that each moon is an instrumental cause ultimately dependent on the sun that makes it an essentially ordered series.

    I'd appreciate your thoughts on these anyway - you have a subtly different take on this stuff to Feser. I've found Feser's approach to these questions confusing, and Aquinas' more so, so I'm interested to see what others make of it. Again, thank you for an excellent post. Blessings, Andrew.

  31. Scott Church says:

    @TY,

    Once again, thanks! I suppose Atheists could shout "non-sequitur" if they like, but non-sequiturs are formal logical fallacies and they would be on the hook to convincingly demonstrate where exactly the alleged fallacy occurs in the proof. Unfortunately, this puts them on the horns of a dilemma--one cannot do that without first being willing to learn the argument and address it point-by-point. As of this writing at least, I've never encountered an Atheist who was willing to do that. To a person, every one of them refused to discuss anything but the knock-off temporal first cause argument (and for the obvious reason ;-) ). As such, I can't say how they would respond to the real one if they were ever forced to actually confront it.

    @Andrew2,

    Thanks so much for your kind words as well! To your points...

    1) The reason the cosmological argument must end in Pure Act is that as I pointed out above, passive potencies necessarily imply contingent dependence. It's meaningless to say that something has a passive capacity for some change without acknowledging the existence of something else that has power over it to bring that about. To say that God has any such potencies is to say that there is at least one thing greater than Him on which He is to some extent dependent. That would render Him no less contingent than the universe--and as such, no better a candidate for the ground of all being--without resorting to a taxicab fallacy.

    2) The same thing applies to multiple grounds of all being. We cannot raise that possibility without addressing the question of how these multiple "grounds" relate to each other, and what determines that. For that matter, the very idea of multiple grounds of all being is arguably self-contradictory. As I mentioned in the essay, it's no coincidence that the Abrahamic religions are all monotheistic.

    3) You are, of course, right... essentially and accidentally ordered causes can be combined. In fact, the examples you gave are the very ones I used. I've kept them in well-separated boxes here only to emphasize the difference between the genuine cosmological argument (which is based on the former), and what I referred to above as the secular "knock-off" one (which is based on the latter). That difference can't be emphasized enough, if for no other reason than that New Atheists in particular will go to almost any lengths to conflate the two, because as noted above, the latter is the only one they can answer.

    4) Regarding your "quibble" with my definition of essentially ordered causality, I agree completely. In fact, I like your definition better than mine. As Aron and I have both argued elsewhere (here and here), simultaneity isn't well-defined in physics anyway, causally or otherwise. But once again, I like to present it this way because for most folks, it makes the distinction with accidentally ordered temporal causality clearer, & that much harder for said New Atheists to segue from.

    Finally, I'm not sure which works of Feser's you found confusing, but if by chance you haven't read it already, I'd recommend The Last Superstition (2010). IMHO, it's the most readable of his works, and makes the ideas I've discussed here most accessible to non-philosophers. The only caveat I'd offer is that it is a polemic. Feser is quite gracious to thoughtful critics and those interested in constructive dialogue. But he has even less patience than I do for folks who believe their right to treat religion with contempt doesn't have to be earned, and he harbors no qualms with taking a razor to Richard Dawkins, Sam Harris, Christopher Hitchens, and other members of the secular illiterati who think fashion and hubris alone trump any responsibility for due diligence. But, as long as you don't mind him intellectually caning folks who ought to know better from time to time, this is the work of his I'd recommend to anyone new to the subject, or those who find it confusing.

    Best to you both! :-)

  32. Derek Smith says:

    Dear Scott,

    This is an excellent post. Thanks for putting all of this together - it was very eye-opening and I'll probably need to read this through several times.

    I do have one question which you mentioned. How does the God Of Classical Theism relate to the Trinity? I don't expect a full reply here (if you are planning to address this in a future post, do say so) but if you do have a link to a good explanation please post it.

    Thanks in Advance.

  33. Scott Church says:

    Hello @Derek, and thanks for the kind words. Actually, I haven't spent as much time thinking through the Trinity from this perspective as I have the topics I discussed here, but I'll offer what I can.

    From the Classical Theistic perspective, I believe most theologians and philosophers would argue that the Father, Son, and Holy Spirit can be thought of as separate views of the same underlying simple "substance" of God, and their relationship with each other as a manifestation of God's knowledge of Himself. At considerable risk of oversimplifying, we might compare this to the way physics can arrive at a system's equations of motion either differentially, or in terms of a static potential or Lagrangian. The two methods aren't independent and can be related to each other, but they provide different vantage points from which the underlying reality can be understood. That analogy is limited of course... I'm certainly not suggesting that the Trinity is some kind of theological formalism, nor that the relationship between the Father, Son, and Holy Spirit can be completely understood that way without any mystery! Rather, it's only meant to illustrate that they might be understood as, in some sense, separate perspectives of the same underlying reality of God as the unchanging simple substance of Classical Theism.

    Others have dealt with all this far better than I can here. For starters, you might try Davies' book The Thought of Thomas Aquinas (1993). Feser has also addressed it here as well as here, here, and here. Hope this helps! :-)

  34. Jonas says:

    Dear Scott,
    Great post, a real pleasure to read. I do wonder, however: Some years ago Aron had a series of posts on whether the universe began to exist or not, and dismissed the BVG-Theorem as inclusive to the question, as in that it doesn't address theories that could potentially be grounded in qunatum gravity theories.
    As Aron gave you the "green light" to have your article uploaded to this blog, has his opinion on this changed?

    Jonas

  35. Scott Church says:

    Hello @Jonas. Thanks for the kind words. Actually, the series of Aron's you referred to is linked in my 3rd footnote, as is his discussion of the BGV theorem in particular (for reference, here and here). I'll let Aron speak to his views on the latter, but here's my take on it. I trust he'll gently correct anything I miss the mark on... :-)

    BGV is what we call a semi-classical theorem, in that it presumes a continuous space-time per classical general relativity apart from the expected impacts of quantum gravity. Compared to earlier "singularity" theorems such as Hawking and Penrose's, its constraints are minimal and broadly-based, making it quite robust and difficult to avoid. A few scenarios have been proposed which conceivably could get around it, but for the most part (IMHO at least) these are contrived at best. The most believable alternatives are the Aguirre-Gratton and Carroll-Chen models. The former requires separating contraction and expansion phases of the universe with a pretty special low-entropy "bounce" that lacks any mechanism to enforce it. The latter is similar, but per Vilenkin, has singularity troubles of its own, and only yields a "past eternal" universe if one conveniently redefines the arrow of time at the minimum contraction point so as to swap past and future. In both cases, the minimum contraction hypersurface, or "bounce", is best understood as a beginning anyway.

    Vilenkin and others have explored multiverse models that might avoid BGV in expanding/contracting inflationary "bubbles" of negative vacuum energy if quantum gravity prevents singularities from occurring there. Aron has worked with Vilenkin and can speak to his views better than I can. But on at least one occasion I'm aware of, he did state that he believes BGV can probably be extended to this case as well. In fact, as long as quantum vacuum energy fluctuations do not become large enough to completely invalidate classical space-time, the theorem is unavoidable.

    The bottom line is this... In principle, BGV can be avoided, and as such it doesn't offer hard proof that the universe had a beginning. However, it's damn difficult to do so, perhaps even within the quantum gravity realm, and I suspect the aforementioned alternatives to it wouldn't be taken nearly as seriously as they are if some Atheist physicists weren't secretly desperate for a cosmology that doesn't smack of a Creator God.

    Best.

  36. kashyap vasavada says:

    Hi Scott,
    This argument of Krauss and others about something from nothing is not only unsound from metaphysical point of view, but also it is misleading at best and dishonest at worst to sell the books! They know very well that what they call nothing is not what common man calls "nothing" i.e. without anything in it! This quantum vacuum has fluctuating quantum fields in it from which the observed universe came out. That is not nothing! Without quantum fields, the model will not work. As everyone agrees, quantum vacuum is a very complicated object. Just having E=0 vacuum does not make it nothing!

  37. Scott Church says:

    Hello @Kashyap, and thanks. Absolutely... Aron has dealt with these, and other claims of Krauss' here, and Luke Barnes has here and here. One of these days I'm gonna have to write about his arguments in more depth myself. :-) Best.

  38. Christopher says:

    Hello,
    Nathan's question about the B-theory made me remember that I have been wanting to ask you guys two questions about time for a while now. First, what do you guys (Aron, Scott or both) think about this paper from George Ellis - https://arxiv.org/ftp/arxiv/papers/1407/1407.7243.pdf He argues for an evolving block universe that is in some ways A-theory like, but is compatible with relativity (or so he claims). I generally ignore proposals about alternate interpretations of time, but Ellis isn't just some guy, so I thought I would ask for your thoughts. Is his proposal viable (even if not preferable to the B-theory) and not in conflict with relativity?
    Second, what about causal set theory? I have heard Fay Dowker describe it like this, "...think of space-time as accreting new space-time atoms in way roughly analogous to a seabed depositing new layers of sediment over time. General relativity yields only a block, but causal sets seem to allow a “becoming,” she said. “The block universe is a static thing — a static picture of the world — whereas this process of becoming is dynamical.” In this view, the passage of time is a fundamental rather than an emergent feature of the cosmos." - https://www.quantamagazine.org/a-debate-over-the-physics-of-time-20160719/ Thank you for all your great posts!

  39. Aron Wall says:

    Christopher,
    I don't agree with any of this: partly for the reason that I think the A-theory is metaphysically absurd as it stands, but also because the proposed modifications of relativity don't seem plausible.

    St. Ellis proposes that we simply slice up time by using the maximal amount of "proper time" since the Big Bang. This makes sense in classical General Relativity, but it's kind of arbitrary, and I doubt it makes sense at the quantum level. In general, the existence of extra structure in spacetime will show up as additional kinds of fields, and although classically you can make sure that these fields don't interact with anything else, quantum mechanically you can't.

    I know Fay Dowker, but I don't agree with her about this. We don't really know yet how to do causal set theory, so people have made as a toy model these models where you randomly add points to spacetime in layers. But these models only involve classical probability theory, and I think they should be working on how to make a true quantum gravity theory (which is hard). Since causal sets have points at spacelike separation, there shouldn't be any notion of absolute simultaneity in the theory, any more than in relativity.

  40. Aron Wall says:

    Jonas,
    First of all, just because I let somebody else post here, doesn't mean I necessarily endorse absolutely everything they write. It only means I think their ideas are worth considering. But, although my take on the BVG theorem hasn't really changed, it's not too dissimilar to what St. Scott said. It provides evidence for a beginning, but there are potential loopholes.

    Derek,
    I said something about how the God of classical theism relates to the Trinity here. Historically, many Christian philosophers have thought that there are some things about God which can be deduced by human reason (basically, Classical Theism) while other aspects of God (like the Trinity) can be known only through revelation. There is, however, one plausibility argument which says that if God is "Love" in his essential nature, then there must be a manifestion of Love within his own essential being, from all eternity. However, it would be presumptuous to go direct from that to the multiplicity of persons in God just from that alone.

    Another way to put this, is that Classical Theism is shared with many other religions, but the Trinity is distinctively Christian. People of all religions have come to the knowledge that there is one God, but having an intimate knowledge of God requires further revelation, and comes for Christians through our knowledge of the Son and Spirit, who reveal what God is really like.

    Andrew,
    I have no idea what a "protective measurement" is either.

    Tom,
    I wrestled a bit with some of these issues in my God and Time series, especially in the 4th (out of 6) post in the series.

    Calhoun,
    You will probably be interested in reading my whole series on God and Time. I don't think that St. Muller is right to say that Divine Simplicity (about which I wrote here) forbids God from having even extrinsic relations to the world. I also think he is wrong to being up the Incarnation of the Son as a counterexample to God's timelessness; Chalcedonian Christians have always admitted that Christ's human nature is in time.

    Nathan,
    You're taking my quote out of context. The commenter was asking me about Euclidean "spacetime", which is completely different from the Lorentzian spaceitme of the real world.

    -----
    With respect to some of the other comments, I would note that St. Scott has not presented a complete argument for Classical Theism. He has only provided some premises from which such arguments have historically been made, and noted that to resist these arguments you need to accept alternative premises (which might not be as plausible). But he is not giving rigorous arguments here.

    I myself am not completely sure that formulating everything in terms of act/potency or between essential/accidental causality is necessarily the best way to go, although these distinctions are at least groping towards something important. My own attempt to render Classical Theism plausible can be found here. The fact is that Classical Theism is something that many philosophers in many cultures have found plausible, even though they did not necessarily arrive at it by the exact same arguments. There is something like "convergent evolution" of ideas here, which makes it more likely that these people were onto something.

  41. Scott Church says:

    @Christopher,

    Regarding causal set theory and Ellis' block universe, I have nothing further to add to Aron's response.

    -----
    To All, I should also note that Aron is correct. Classical Theism is far too big a subject for one post, and so are the differing schools of thought regarding causality, act, and potency. I had no illusions about being able to offer a full treatment of any of these topics. My only focus here was a discussion of why god hypothesis arguments don't hold any water, and it was only to that end that I introduced them. We are talking about God, after all. Far be it from me to imagine that I could wrap up such recondite topics in one pretty package, with glitter tape and a big pink bow! :-)

    But that said, the field is a fascinating one and I hope I've whet a few appetites. If so, those of you who want to jump in feet first could do a lot worse than starting with some of my cited sources and links.

  42. Nathan says:

    Hi Aron,
    Thanks for the reply. So, just to clarify, you don't deny the reality of change? You say, "The commenter was asking me about Euclidean "spacetime", which is completely different from the Lorentzian spaceitme of the real world" So it is safe to say that while things like duration might be considered relational properties in modern physics, SR and/or GR don't show that all motion and change are illusions? Sorry for my confusion on this.

  43. Aron Wall says:

    Nathan,
    What do you mean by "change"? I think all times are equally real. So things don't really "pass out of existence", but they do vary in a nontrivial way on the time coordinate.

    But I also believe that causality is a real feature of the world, that each event that happens is caused to exist by (some subset of) the things in its past. It is this feature, causality, which makes a lot more sense in a Lorentzian world than in a Euclidean world.

  44. Andrew says:

    Alright, thanks Aron & Scott, maybe what I was reading from was a little dodgy.

  45. Jonas says:

    Dear Aron,

    Thanks for answering me so kindly.
    Now I have another question, If I may (both You and Scott are obviously welcome to answer):
    The atheist could state that we only know causes from our daily life, but those are merely transformations of matter from one form into another. We never actually witness true creation, where something entirely new is created, and there is no evidence suggesting that such a thing happens/exists. We only know change, creation and cause from within the universe, no reason to apply it to the universe as a whole.

    What would your answer be?

    Jonas

  46. Scott Church says:

    Hello @Jonas,

    As a matter of fact, some Atheists have made that argument. Sean Carroll, for example, made a similar one in his 2014 debate with William Lane Craig. IMHO, there are at least three fundamental problems with it.

    First, the claim that we never witness "true creation" has no teeth because it can't be proven, even in principle. Suppose a rabbit were to appear before us out of thin air. How would we know that it actually began to exist at that moment? How would we know it hadn't been "beamed" to that location from elsewhere via some as-yet undiscovered wormhole or teleportation physics? As long as these possibilities can't be ruled out, this is just an assertion of blind faith no different than what Atheists routinely accuse religion of. Even if it wasn't, it's still irrelevant. Not only is the Thomistic Cosmological argument I've presented here not dependent on it, it presumes it to be false. As we saw, the argument begins with Parmenides' assertion ex nihilo nihil fit--out of nothing comes nothing.

    Second, when it gets right down to it many, if not most Atheists assert that we do witness true creation from nothing. The arguments range from bad to downright laughable, but that's beside the point. For one to assert that we have no evidence of "true creation" for events within the universe, but appeal to it for the universe as a whole is to commit a taxicab fallacy. An Atheist could respond by arguing that attributing causality to the universe as a whole from its parts commits a fallacy of composition, but I've already addressed that in previous comments.

    But the biggest problem with it is that it doesn't address the real issue. It must be remembered that as time-bound creatures with five senses, our experience of the universe is one of "flowing through" history along our own world-lines. But from the standpoint of physics, the entire history of the universe exists as a single space-time manifold for which no event at any particular time and place is any more real than any other. What we experience as "change" is just monotonic variations in the equations of state and/or dynamics of matter from event to event along world-lines, with causes and effects being separated from each other via a time asymmetry resulting from the Second Law of Thermodynamics. The core issue for Theists and Atheists here boils down to two questions: Why is there something rather than nothing? and "Why does this particular universe exist, with its unique combination of parameters, equations of state/dynamics, and world-line histories, rather than an infinite number of other equally possible ones?

    These questions remain regardless of whether we experience change as the flow of being into becoming or monotonic variation along our world-lines, or whether any given world-line/s has a singular beginning or not, and they demand answers. As I've shown above, Theists can address them starting only with the observed facts of being and becoming (which I've expressed here in terms of the Thomistic concepts of act and potency), and essentially-ordered vs. accidentally-ordered causality, whereas Atheists must complicate matters with layer upon layer of ad-hoc baggage.

    Best. :-)

  47. Aron Wall says:

    Jonas,
    You are quite correct that no physical being can cause another to exist in the exact same sense that God creates things. God gives things (not just in the distant past, but in the present) their being, whereas other things can only, as you say, transform beings from one form to another, and rearrange them into different configurations. And they only even have the power to do this, because God gives them that power.

    In the traditional Scholastic way of looking at these arguments, God's "creation" and other attributes are held to be analogous to ours, where analogy is something in between "univocal" (word is applied in the exact same sense) and "equivocal" (word is applied with a completely different meaning). Analogy means that the word is applied with a related, but still qualitatively different, meaning: e.g. healthy food vs. healthy people; free beer vs. free speech. I've argued, somewhat similarly, that we can only really understand God by means of metaphorical inference from created things.

    Since this is so, no possible argument for the existence of God can be completely 100% logically watertight, because whatever principle of causation you wish to consider, the atheist can always say that it applies only in ordinary, natural contexts and does not allow the leap to a corresponding transcendental cause. However, this move comes at a steep price, since it implies that many important features of the world have no explanation at all! (The most intellectually savvy atheists, like Sean Carroll or Bertrand Russell, are quite frank about this.) Whereas Theism provides a unified explanation for a number of different (otherwise puzzling) aspects of reality. The flip side is that many Naturalists generally try to deflate or elminate these aspects of the world (like causation or ethics or consciousness or math...) as much as possible because they don't quite make sense in a purely physical universe, but of course they can never succeed entirely, since human thought presupposes the validity of such categories.

    If you want more information about how I look at this, you can read my Fundamental Reality series (if you have not already done so).

  48. Jonas says:

    Aron and Scott,

    I am really grateful for the opportunity to have you two answering me. I would have no idea who to ask those questions, as I dont know anybody with any knowledge in the relevant fields. The internet really proves to be a tool for communication!
    I am obviously grateful for you to answer me in the first place as well ;). I think I shortly dived into it at one point. I was sure that I had my question answered some other time, but I couldnt remember where. Maybe it was in your link all along?

    I suppose whenever I have similar problems, maybe in the future, I will bother you again. Sorry for that!

    Jonas

  49. Scott Church says:

    @Jonas,

    I'm sure I speak for Aron as well when I say the pleasure is ours! It gives me no end of joy to think that my humble thoughts might be helping someone on their spiritual journey. As a college student (back in the dark ages) I struggled horribly over many of these things. Had Aron been around then, his wisdom on these matters would've been the very face of Christ to me. It has been, of course. But it would've been all the more then.

    I honestly can't think of any greater gift from God than to be able to do whatever I can, little or great, to return the favor and help others with the same struggles. If you have similar problems in the future, by all means, don't feel sorry... reach out! I live for the joy of getting a chance to help with this sort of thing if I can!

    Best. :-)

  50. Georgios Bosch says:

    "That cannot be done in a vacuum without committing oneself to some, or all, of the following counter-axioms;"

    This statement, and what follows it, is completely wrong. It presumes that the arguments theists have come up with to prove what is really a feeling are somehow watertight, rigorous arguments, which follow in a mathematically precise manner from a set of axioms. Nothing could be further from the truth. The notions of "contingency", so prized by theists of a more philosophical bent, aren't even sufficiently well-defined to draw any sort of conclusion from. In practice they're nothing more than a semi-sophisticated way of begging the question, by defining not-contingent as "god" and "contingent" as "everything else". Not very convincing.

    Cosmological arguments and its numerous variants typically operate in this way. Grab some element presumed to have been sufficiently argued, such as the existence of a first cause, and identify that first cause as "god". Whether that first cause actually does have the properties the theist requires of a god (and there seem to be as many variants of these as there are theists) is left as an exercise to the reader. That said, even the existence of the first cause is an unjustified assumption, as we can see for example in the celebrated Five Ways of Aquinas:

    "(...)whatever is in motion must be put in motion by another (...) then this also must needs be put in motion by another, and that by another again. But this cannot go on to infinity, because then there would be no first mover, and, consequently, no other mover; seeing that subsequent movers move only inasmuch as they are put in motion by the first mover..."

    There must be a first mover, because otherwise there would be no first mover. Again, hardly convincing. Incidentally, and lest I forget, claims of uniqueness are on even shakier ground than claims of existence, and once again amount to little more than semantic wordplay rather than rigorous philosophy. There's nothing whatsoever inherently contradictory about the assertion that the universe might have multiple causes, and in fact it doesn't even seem a priori inconsistent that the universe might have an uncountable set of causes. I can't envision what a continuum of gods might look like, but what physics should have taught you is that your ability to envision something is entirely irrelevant.

    The arguments for uniqueness rest on a particular choice of description, favored by monotheists, but it is a choice. It does not have to be a correct, useful, or illuminating description. It's really an opinion, just like defining god to be a first cause in the first place was merely opinion. Choosing to call something "god" tells you nothing about that thing. It says something about you. It says that you choose to worship whatever that thing is, which is once again a choice. Even if every single one of the arguments you presented were correct, it would still not get past this fundamental barrier. You can choose to call this first cause a god and worship it, or you can call it something else and think of it as part of the description of the natural world. It's a choice. Neither choice is obviously better than the other, but you can't deny the choice, and that's what you did when you created a list of various affirmative positions that atheists must supposedly take.

    While it may vex you that atheists refuse the burden of proof, that doesn't mean they're wrong. If they're not making the claim that there is no god, but rather, simply remain unconvinced that there is one, they don't owe you a single shred of evidence in support of their position. All they need do is poke holes in the evidence you provided in support of yours.

  51. Scott Church says:

    @Georgios,

    Where to begin...? Let me start be addressing a few of your comments inline;

    "The notions of "contingency", so prized by theists of a more philosophical bent, aren't even sufficiently well-defined to draw any sort of conclusion from. In practice they're nothing more than a semi-sophisticated way of begging the question, by defining not-contingent as 'god' and 'contingent' as 'everything else'."

    The contingency of the universe is simply the observation that it undergoes change from state to state, and these states conceivably could've been different. When I woke up this morning it was 48 degs. F outside my back door, but as I type these words it's risen to 57 degs. F. The fine structure and cosmological constants have well-known values that as of this writing, are under no known theoretical constraints and could've assumed an infinite number of other values without violating any law of physics. This morning I prepared and ate oatmeal for breakfast, which is now in my stomach being converted into energy. Etc. etc. etc.

    I can't speak for you of course, but I have no trouble at all envisioning how any of these things, and countless others, could've been different. It could be raining and cold here rather than a nice Indian summer day. I could've had an egg sandwich for breakfast instead of oatmeal. And I assure you that 48 degs. F is a lower temperature than 57 degs. F. Every one of these represents a change in some part of the universe, and none of them just happened by magic. Various actual state/s of the physical world interacted with others in ways that brought about (actualized) some, but not all specific states they potentially could be in but hadn't yet realized.

    Which of these "notions" isn't "sufficiently well-defined to draw any sort of conclusion from"? If you're aware of one or more logical points and/or theoretical constraints that make it virtually impossible even in principle for the universe or laws of physics to be any different than they are, or for that matter, even for me to have had an egg sandwich for breakfast this morning, then by all means, publish it... and go collect your Nobel and Templeton prizes.

    "[E]ven the existence of the first cause is an unjustified assumption, as we can see for example in the celebrated Five Ways of Aquinas... There must be a first mover, because otherwise there would be no first mover."

    I see that you (or whoever you might've borrowed this quote from) have quoted Aquinas rather selectively. It seems a more thorough look at what he actually said is in order;

    It is certain, and evident to our senses, that in the world some things are in motion. Now whatever is in motion is put in motion by another, for nothing can be in motion except it is in potentiality to that towards which it is in motion; whereas a thing moves inasmuch as it is in act. For motion is nothing else than the reduction of something from potentiality to actuality. But nothing can be reduced from potentiality to actuality, except by something in a state of actuality. Thus that which is actually hot, as fire, makes wood, which is potentially hot, to be actually hot, and thereby moves and changes it. Now it is not possible that the same thing should be at once in actuality and potentiality in the same respect, but only in different respects. For what is actually hot cannot simultaneously be potentially hot; but it is simultaneously potentially cold. It is therefore impossible that in the same respect and in the same way a thing should be both mover and moved, i.e. that it should move itself. Therefore, whatever is in motion must be put in motion by another. If that by which it is put in motion be itself put in motion, then this also must needs be put in motion by another, and that by another again. But this cannot go on to infinity, because then there would be no first mover, and, consequently, no other mover; seeing that subsequent movers move only inasmuch as they are put in motion by the first mover; as the staff moves only because it is put in motion by the hand. Therefore it is necessary to arrive at a first mover, put in motion by no other; and this everyone understands to be God.

    - Summa Theologiae, Question 2, Article 3 (My Emphasis)

    What Aquinas is actually saying is that there must be a first mover because motion (change) is the reduction of something from potentiality to actuality by something in a state of actuality, and it isn't possible for anything to be at once in actuality and potentiality in the same respect, and as such, able to move itself. In other words, there's no such thing as magic and bootstrapped changes. My neighborhood didn't warm itself from an actual state of 48 degs. F to a potential one of 57 degs. F this morning. An actual star called the sun warmed it. And to the extent that any thing has potentials that can be externally realized by something else (i.e. - isn't purely actual), it's dependent on something else more fundamental than it for part, or all of its actuality and as such, isn't a fundamental reality that can provide a bedrock answer for why there is something rather than nothing.

    Seems pretty damn reasonable to me. If it doesn't to you, you're more than welcome to demonstrate magic to us or pick yourself up by the bootstraps. Proper context is a bitch, ain't it? :-)

    "There's nothing whatsoever inherently contradictory about the assertion that the universe might have multiple causes, and in fact, it doesn't even seem a priori inconsistent that the universe might have an uncountable set of causes. I can't envision what a continuum of gods might look like, but what physics should have taught you is that your ability to envision something is entirely irrelevant."

    Agreed. There's also nothing inherently contradictory about a universe caused by an infinite continuum of transvestite goth fairies with nipple clips and flame throwers. But why on earth would I want to "envision" one? A diversity of gods, fairies, causes, or anything else implies a diversity of traits that must be distinguished from each other in potentially realizable ways, which only raises the causal regress issues noted above that are avoided by the pure act God of Classical Theism. As a Classical Theist, I have no need to make such a mess of my worldview to avoid conclusions that are unfashionable or threatening to my beliefs.

    You see, physics did teach me that my ability to envision something is irrelevant... which is precisely why I don't bother envisioning "a continuum of gods", "multiple causes", fairies, or any other frivolities ripe for Occam's Razor when I have exactly zero need to. As a matter of fact, it was none other than David Hume who made what we can imagine, or envision, the basis for denying the causality concepts I've presented here. According to him, our ability to envision things appearing from nothing and returning to it proves that there's no basis for considering streams of events, or percepts, as anything other than "loose and separate" (Hume, 2017). His thought on the matter informed entire generations of modern Atheist thinkers after him, and in fact became the basis for several of the Atheistic counter-axioms I listed above.

    Essentially, what you've done here is incorrectly accuse me of relying on one of your team's favorite arguments and then faulted me for it. That is what's known as a self-refuting claim.

    "The arguments for uniqueness rest on a particular choice of description, favored by monotheists, but it is a choice. It does not have to be a correct, useful, or illuminating description. It's really an opinion, just like defining god to be a first cause in the first place was merely opinion. Choosing to call something "god" tells you nothing about that thing. It says something about you."

    Um, no they do not. Had you bothered to actually read my essay carefully (which it's becoming increasingly apparent that you did not) you would've seen that the cosmological argument presented here does not end with any definition of "god". It ends with something that is pure act--an eternal, unchanging, omnipotent simple substance possessing omniscience, intellect and will. Aristotle referred to this pure act as the unmoved mover, not "god". Later thinkers, particularly the Scholastics, saw a striking similarity between this unmoved mover and the God of the Bible and Quran they'd already discovered via their independent theological studies, and thus concluded that the two were one and the same. Hence, Aquinas' comment in the complete quote above that we understand the unmoved mover to be God (capital "G"). At no point whatsoever did any attempt to "define" God, or any "god" enter into the discussion until the Enlightenment... when Atheist thinkers found themselves in need of a version of the cosmological argument they could actually answer and set out to invent one.

    If I were to, say, visit the seashore, and conclude based on a variety of observations and arguments that there existed something that was alive and cold-blooded, breathed via gills, and had pectoral, ventral, dorsal, anal, and caudal fins, I wouldn't arbitrarily "choose" a definition for it. I'd note the striking similarity it had to the infraclass of Teleosts (ray-finned fishes) and consistent with other independent studies of my own, conclude that it was a fish. You are correct though... doing so certainly would "say something about [me]".

    It would say that in addition to visiting the beach, I'd paid attention in my biology classes and passed them. ;-)

    "While it may vex you that atheists refuse the burden of proof, that doesn't mean they're wrong. If they're not making the claim that there is no god, but rather, simply remain unconvinced that there is one, they don't owe you a single shred of evidence in support of their position. All they need do is poke holes in the evidence you provided in support of yours."

    Again, agreed. Refusing the burden of proof certainly doesn't make Atheists wrong. It does, however, tell me a great deal about them--at least, as you've defined them here.

    For starters, those who aren't making the claim that there is no "god" are Agnostics, not Atheists. As you rightly claim, remaining unconvinced of God's existence doesn't obligate them to a burden of proof, and as such, I haven't addressed them here. Atheists, however, are another matter. They do claim that there is no god... period. As I've shown above, that is a positive claim about the nature of reality that does carry with it a burden of proof. Those who refuse one before they've even bothered to open a dictionary and properly identify their own damn stance are at best intellectually lazy, and at worst incompetent and/or dishonest. At a bare minimum, rejecting belief in God requires at least some due diligence--an understanding of what exactly is being rejected, and a reasonable standard of proof that isn't being met by it. I've yet to meet a single Atheist who could provide me with either, much less both, or for that matter, was even willing to allow peer-reviewed science, philosophy, or comparative religion into any discussion. It's damn difficult to "poke holes" in "evidence" one has neither a functional understanding of nor even a willingness to learn.

    Needless to say, real Atheist thinkers will have none of this sloppiness. As I noted above, those of any repute throughout history have spent their lives laboring to produce rigorous, properly thought-out philosophies of Materialism based on most, or all of the Atheistic counter-axioms I listed above. I suspect that's because they know something about the true nature of "question begging" that has so far eluded you.

    ---------------------

    I do appreciate your feedback Georgios, but the points you've addressed bear no resemblance whatsoever to anything that's been presented here. You seem to be responding instead to what I referred to above as the knock-off temporal cause argument. New Atheists will go to almost any lengths to steer discussions like this one to that argument because they cannot credibly respond to the one that was actually presented. The mere fact that you describe "god" as a defined first cause, and the arguments given as "evidence" shows that you've completely missed the fundamental point of the latter, and of this entire essay--namely, that the God of Classical Theism (note the capital "G") is a formally derived conclusion, not a hypothesis amenable to a properly evidence-based scientific approach.

    Perhaps this is presumptuous of me, and if so, I apologize in advance. But it would appear that you made no attempt to read any of this carefully before commenting, but simply parroted arguments you found in some popular books and/or chat rooms. Even your reference to Aquinas' "celebrated" five ways seems odd. No one trained in the relevant fields speaks that way of this topic, any more than physicists speak of Newton's "celebrated" laws of mechanics. Both are fundamental to the subjects, and no one properly versed in them would waste time commenting on their obvious "fame". This is, however, the sort of comment we'd expect in popular works by people like Richard Dawkins, Sam Harris, and others who are illiterate enough in these subjects to think that pointing out how "celebrated" they are is some kind of revelation.

    If I may, I would encourage you to read this essay and the comment thread again... this time carefully... and review the linked and cited source material as well. Doing so won't necessarily compel you to agree with me of course. But if nothing else, it will at least make your responses informed, and as such, potentially constructive.

    Best.

  52. Mactoul says:

    Scott Church,
    Contingency is not an empirical fact but something metaphysical. As you write:
    "The contingency of the universe is simply the observation that it undergoes change from state to state, and these states conceivably could've been different."

    A fatalist could say that your conceivability is mistaken. If it is 57 today, it was bound to be so, by all the laws of physics, it could not have been otherwise.

  53. Scott Church says:

    Hello @Mactoul,

    As you said, the contingency of the universe is a metaphysical statement rather than an empirical one--I stated as much in the first of my metaphysical axioms above. But like the rest of those axioms, it wasn't made in an empirical vacuum. Unless one is doing some serious drugs, being and becoming--the existence of an ever-changing universe--is as hard an empirical fact as there is. And not only is there virtually nothing in any known or foreseeable laws of physics to suggest that its existence and evolution couldn't have assumed an infinite number of alternative forms, there's little hope that an irreducibly indeterminate universe can even be avoided, much less evidence for one in which "it was bound to be [57 today]". Your ability to even postulate if this is so in the first place presumes that it's entirely possible that the universe could've been otherwise.

    Fatalists can call my "conceivability" mistaken if they like. But if they want to be taken seriously they'll need to demonstrate that considering "if it is 57 today..." to be impossible--even meaningless--is simpler, more to the point, and more consistent with the laws of physics and our experience than not doing so. That Sir, is a tall order! If they succeed, they too can go collect their Nobel and Templeton prizes and retire to the lecture circuit. Indeed, if they're right, they'll have no choice but to do so. :-)

  54. Mactoul says:

    Contingency is opposed to necessity. Thus an ever-changing universe is no evidence for contingency since the same changes could have happened by necessity.
    Contingency is an intuition we have that the things could be otherwise than they actually are. As such, it pertains to the things and events. But when you talk of the contingency of the universe itself, there is no such intuition. You write
    "The universe is contingent. Its essential nature, or form (and that of everything in it) is separate from its existence"

    It is unclear as what is meant by "essential nature or form" of the universe. Do you mean the fundamental laws of physics?

  55. Scott Church says:

    @Mactoul,

    I'm not sure what your point is here. Forgive me, but you're not making a lot of sense.

    Obviously, contingency is opposed to necessity. But... you do realize, don't you, that by itself, the mere fact that multiple possibilities exist doesn't mean all are equally likely and no evidence whatsoever favors any of them, right? By definition, the universe is just the sum total of "the things and events" it contains. So... if those things and events are contingent, how exactly does that not lead to the universe being contingent? This is like arguing that a set of wooden red blocks isn't red or wooden. What you're doing here is committing a Taxicab fallacy. You seem to think we can talk about the universe separate from everything in it.

    Nor am I sure what you think is unclear about its essential nature or form. Look around and you'll see that all those "things and events" are identifiable and comprehensible. Gravitating masses attract; the Higgs boson has different spin properties than the electron and a different mass; the sun is hotter than the earth; I am not Aron Wall; Aron Wall is not you... etc. etc. etc. All of this obeys rational laws from which at least some (but not all) predictions can be made--what Eugene Wigner referred to as "the unreasonable effectiveness of mathematics". Broadly speaking, there are two ways we might account for this comprehensible order;

    a) Nature is comprehensible purely by coincidence and could evaporate into chaos at any moment for literally no reason whatsoever. Or,

    b) Nature is comprehensible because it has a comprehensible underlying character and order--the essential nature or form of things.

    Those two possibilities are not equally likely-- which is why you'll be hard-pressed to find any physicist or metaphysician who's confused by the latter.

    If I may, let me just leave you with this... I do agree that contingency and necessity are both at least possible, as are an underlying order in the nature of things and chaos. In both cases, you're more than welcome to embrace the latter if you like. But if I were you, I wouldn't get my wallet out and take those odds to a bookie. ;-)

    Best.

  56. Mactoul says:

    Scott Church,
    You rather oddly define contingency as
    "The universe is contingent. Its essential nature, or form (and that of everything in it) is separate from its existence."
    Now, the individual things have forms but how can universe, being just a collection of things, can be said to have a form or essential nature? This is what I keep asking but without getting any response. What is the form or essential nature of the universe?

    There are other interesting questions that might be asked. For instance, how is this Thomistic picture--form, potency, essential nature etc etc harmonize with the quantum mechanical picture of the same universe where, according to you, everything is composed of wave functions?

  57. Scott Church says:

    @Mactoul, with all due respect, you have been given a response. You don't seem to be paying attention to it. How can the individual things have forms but the universe, being just a collection of things, have a form or essential nature, you ask?

    Simple... the form or essential nature of the collection of things known as the universe is just the sum of the forms or essential natures of the things in the collection.

    It's really not that difficult. If I show you a collection of things--100 red blocks made out of wood for instance--then that collection of things is... [wait for it]... red and made out of wood. Redness and wooden composition are part of the essential nature of that collection of things because the essential natures of the things in it are wooden and red. And inasmuch as those blocks are ultimately composed of subatomic particles, as is the collection they're all part of, then there will be a many-body wave function that describes both, and it too will be part of their essential nature--which is precisely why quantum mechanics is a successful theory that actually tells us something about the essential nature of the collection of things known as the universe.

    Forgive me, I realize this might sound rude. But if you honestly can't grasp the difference between asking what a thing or collection of things is, including the universe, and whether it is, I just can't dumb it down any further and I'm not going to keep trying to. You're more than entitled to disagree with me of course. But wading deeper into a quicksand of endless hair-splitting where even the simplest of concepts is swallowed won't benefit anyone, including you. Especially since from the looks of it, you're the only one here who seems to find any of this "odd". But that said, thank you for your thoughts. It may not sound like it, but I do appreciate them! :-)

  58. Mactoul says:

    Scott Church,
    Your example of a hundred red blocks indicates that you are using the technical word "form" in a non-standard way. Your modern physics keeps on intruding on ancient metaphysics. Let me clarify.

    "-100 red blocks made out of wood for instance--then that collection of things is... [wait for it]... red and made out of wood."

    The collection is red and wooden but the form of the thing made is independent of it being wooden and red. It could be a chair, a wall, a table etc or just a random heap of blocks.
    So, while the thing is red and wooden, its form is not "red and wooden". Modern physics is concerned with the material cause--what the thing is made up of, but the "form" relates to what the thing "is".

    Your assertion
    "the form or essential nature of the collection of things known as the universe is just the sum of the forms or essential natures of the things in the collection."

    requires that we know the sense in which the "forms" can be summed. I don't know the rules of this operation which I haven't heard of previously. You may care to elucidate.

    Mu further question was "what is the form of the universe?"
    I make precise the definition of the universe which I take from Fr Jaki--"universe is the totality of consistently interacting things."
    Now a totality is not guaranteed to have a form, to be precise "a form of its own". For instance, a heap of sand does not have a form or essential nature--it is pretty formless. The individual grains of sand do have form but not the heap. Quantum mechanics does not enter. The heap-ness of the heap of sand is not precise enough for us to endow an essential nature to the heap.
    So, my question of whether the universe is totality such as a sand heap or something else.

    A possible answer to the question of the form of universe be made pointing to its properties--whether it is hyperbolic spacetime, or euclidean, i.e. what is the solution of general relativity equations the universe corresponds with.

  59. Jonas says:

    Hey Scott & Aron,

    Ex nihilo nihil fit. How would one defend this, on the face of it, logically necessary statement? (I would have never thought Id ask this, to be honest, but here I am) What about the zero-energy universe? It being "created" would still require something outside of itself to make it happen, right? Just that no "material" is needed for that, that it doesnt need to be made out of something else.
    Also, what would your argument for the impossibility of an infinite regress be?

    @Mactoul

    On your more original comment: Every empirical fact is also metaphysical, and vice versa. The metaphysical view of monistic materialism came about through the empirical observation of an external world seemingly made out of matter.

    Also, lets take Leibnizs argument from contingency, based on the Principle of Sufficient Reason.
    The PSR states that everything has a reason to explain it. That reasons comes either from outside of the thing question, then it is contingent, or from inside itself, out of its own necessity.

    We observe that nature could have been absolutly different, or entirely non-existent (why exists something rather than nothing?). There has to be a reason for why nature is the way it is (Natural laws, by the way, are only the description of the way that natural things behave), and why it exists at all. Nature doesnt explain its own existence though. Therefore, there has to be something outside of nature on which its existence is grounded. That something has to be necessary being

  60. Mactoul says:

    Jonas,
    My problem is not with the conclusion but with the form of the argument which I think is very inadequate.
    There is too much loose language concerning "universe".
    For instance you write
    "We observe that nature could have been absolutely different or non existent."
    Now
    i) By "nature" I suppose you mean the great interlocking system of cause and effects as CS Lewis calls in the Miracles.
    ii) Which observation tells you that this nature could have been absolutely different?

  61. Scott Church says:

    Hi @Jonas,

    Thank you for yet another round of thought-provoking questions! I believe we can address all three by starting with the first and its relationship to the contingency of the universe.

    There are several ways one could defend Ex Nihilo Nihil Fit, but perhaps the best is to just name rejection of it for what it is, and watch what its critics do with that. Ex Nihilo Nihil Fit ("out of nothing comes nothing") is simply the claim that non-existent things cannot acquire existence on their own via non-existent causes. Denying that is essentially to embrace belief in magic. Inasmuch as the best arguments for God's existence begin with it, this puts Atheists in the unenviable position of having to credibly defend magic and/or a denial of causality, and that after having proclaimed themselves ambassadors for science and reason. That's damn difficult to do without resorting to considerable equivocation and sophistry. To wit...

    Earlier I mentioned a New Atheist friend and colleague to whom I once presented some of the arguments in this essay. His response was that it's more reasonable to believe the universe originated from some "unknown change" at the big bang. I pointed out that he'd misunderstood the cosmological argument by conflating essentially-ordered passive causality with temporally-ordered active causality, to which he responded that "most people" understand it the way he does (a textbook bandwagon fallacy). When I asked what sort of "change" could create itself as well as everything else, he became evasive and changed the subject.

    So... the universe exists because a non-existent "change" he can't even identify, much less theoretically formulate, changed non-existence into existence. :-)

    But wait...! What about the "zero energy" universe? According to New Atheist physicist Lawrence Krauss, "nothing" has "no energy" so the universe originated from quantum fluctuations in a zero-energy vacuum. Never mind whether the universe actually has zero-energy or not--a point which is at best debatable, and at worst a muddle. To anyone remotely familiar with logic and the English language, "nothing" is not a noun. It's the absence of nouns and whatever associated properties they might have, including energy. It's one thing to say that nothing has no energy. It's another thing altogether to say that "it" has an energy, and that energy is equal to zero joules/m^3.

    But who knows? Maybe if we equivocate enough we can gaslight people into missing such subtleties. In his book A Universe from Nothing Krauss tells us that;

    "[N]othing is a physical concept because it's the absence of something, and something is a physical concept."

    In other words, if something is a noun then its absence is one too. He goes on to tell us that in an expanding universe “nothingness would reign supreme”, and this "nothingness" is "much more complicated than we would've imagined otherwise".

    So... the universe exists because there are some things which are things, and some things which are not things so they're nothing even though they're some things.

    Who's on first...? :-)

    But wait...! What about an infinite regress of causes? How do we know that the universe isn't caused by an endless chain of caused things without a first cause? Again, it must be reiterated that what's on the table is an essentially-ordered hierarchy of passive causes. Consider my previous freight train example. A freight car only has the power to pull other cars if it's given that power by being pulled itself, and... [wait for it]... this is every bit as true of a train of freight cars, including one that's infinitely long or connected to itself in a circle. Unless there's at least one car in it that possesses the power to pull cars actively in itself, the train cannot move. Now the universe is just the sum total of its physical contents, all of which possess passive properties, including their existence, so the same is true of it.

    So... the universe exists because adding endlessly to an infinitely long chain of passively-caused things like freight cars magically transforms it into an actively-caused one with powers of self-creation. :-)

    And so on, and so on... People like Krauss and my friend insist that this sort of thing is more rational than belief in the purely actual creator God presented in this essay and my sources. Perhaps to them, it is. But to everyone else, their idea of "rational" speaks for itself rather loudly.

    In the end, while there's no shortage of ways to defend Ex Nihilo Nihil Fit, it's easier, and more entertaining, to just call out its rejection by name, then grab a drink and relax in a lawn chair while its secular critics flop around in the kiddie pool trying to get out of the straightjacket they've made for themselves.

  62. Mactoul says:

    The philosophy of quantum mechanics essentially rejects Ex Nihilo Nihil Fit Even worse, it rejects distinction between being and non-being, thus rendering Ex Nihilo Nihil Fit meaningless.
    As Stanley Jaki puts it.
    "The science of quantum mechanics states only the impossibility of perfect accuracy in measurements. The philosophy of quantum mechanics states ultimately the impossibility of distinguishing between material and non-material and even between being and non-being. "

    The time-energy uncertainty relation "del E*del t>h/pi" interpreted as leading to creation of particle-antiparticle pairs--is the beginning of theft of being. And once one is used to steal on small level, it is easy to progress to grand-scale thefts of the entire universes/

    [Corrected typos. The uncertainty relation should be \Delta E \Delta t \ge \hbar/2---AW]

  63. Jonas says:

    Aaaaand here I am, again! :) Thank you again, for answering Scott!

    Short question to the fine-tuning argument this time, which is what I take to be more "technical" than philosophical: How do we know the constants could have been different? I see nothing necessary about them in their current form at all, but how would one answer this question from a physicist's perspective? Or at least philosophy of science perspective?

    @Mactoul

    Hello again,

    i) Yes, I suppose that's roughly what I mean by it. The cumulation of all contingent things we experience in the material world is what we call "universe", or rather "nature". I've never read C.S. Lewis though, not even the Narnia books
    ii) If the single objects building the universe were not contingent, then they'd be necessary, meaning they could not have been any different or non-existent. Yet, they rely on things other than themselves to explain their existence/happening. The objects exist and are not being a se, therefore their existence is contingent on something that is being a se.

    Does such a thing as chance or choice exist according to you?
    Tomorrow, I assert, you're going to wake up and take the blue shirt to wear. Could you not, absolutely impossible so, have taken they white shirt rather than the blue one?

    iii) I'm not into the philosophy of quantum mechanics. Yet I do have to wonder: How did they establish that rejection? On what grounds? The only way they could do so would be if they were to redefine "nothing" into "something", be it the quantum vacuum or information or whatever.
    I guess Aron would be the one to comment on "the impossibility of distinguishing between material and non-material and even between being and non-being".

    I couldnt find anything on Stanley Jai on the fly. Maybe you could provide a link on him, or something?

    [Corrected typos. You probably couldn't find Stanley Jai because it was a typo for Jaki. But please note I am not endorsing Jaki's interpretation of QM. I haven't read his book, but the quote above seems to me to be nonsense.---AW]

  64. Mactoul says:

    @Jonas,
    "Does such a thing as chance or choice exist according to you?"
    Well, choices can only be made by beings having free wills and are, in any case, entirely outside the province of physics.
    Whereas, "chance" merely means ignorance in classical mechanics, while the "chance" as it appears in quantum mechanics has never been harmonised with Thomistic metaphysics of contingency.

    My questions stands. Leaving free-willed us aside, the lifeless things are what they are. The cascade of events they produce and take part it, is what it is. How could it be different?
    Given the initial conditions and the laws of physics, the cascade of events flows uniquely. Appealing to quantum mechanics does not help you, for the quantum mechanical "chance" is unintelligible and contradicts PSR.

  65. Mactoul says:

    "If the single objects building the universe were not contingent, then they'd be necessary, "

    Agree but the question is whether this makes sense in quantum mechanics which as Aron Wall and Scott Church would maintain is the true description of affairs.
    Are there "single objects" in quantum mechanics? As Scott Church has written, there are quantum fields and frankly the physicists are shy of explaining exactly how do we get everyday objects out of the quantum fields.

  66. Scott Church says:

    Hello @Jonas,

    Once again, great questions! How do we know the constants could have been different...? For exactly the reason you stated--there's nothing necessary about them in their current form at all. That is the perspective of physics. Take the cosmological constant for instance. As of this writing its observed value is (to the best of my knowledge at least) 1.50 \pm 0.02 \: x 10^{-25} \: kg/m^{3}, or in dimensionless units, on the order of 10^{122}. No known law of physics requires this value, and there isn't a reason in the world why it couldn't have been, say, 10^{125} or 10^{119}. Had it assumed one of these values instead, the universe would've either recollapsed on itself fractions of a second after the big bang or expanded into a vast inflating emptiness with no more than a few subatomic particles and photons per cubic light year... and we wouldn't be here. There isn't a shred of credible evidence, of any conceivable kind whatsoever, to suggest that known or foreseeable physics deterministically constrains it to the value we observe. In the absence of any, denying that the universe is contingent in at least this respect (not to mention an infinite list of similar examples) are just blind assertions that are no different than belief in the Tooth Fairy, and every bit as subject to Occam's Razor.

    Regarding the philosophy of quantum mechanics, I assume your question to Mactoul as to how physicists "establish that rejection" is in reference to his quote of Fr. Stanley Jaki's (Sept. 28, 2017);

    "The science of quantum mechanics states only the impossibility of perfect accuracy in measurements. The philosophy of quantum mechanics states ultimately the impossibility of distinguishing between material and non-material and even between being and non-being."

    With all due respect to Jaki, as Aron said, every word of that is utter nonsense. While it is true that QM begins with indeterminacy in observed measurements, it hardly ends there. And unless I'm misunderstanding Jaki on this point, taken at face value this statement appears to be an endorsement of hidden variables theories, which theoretically and observationally are in deep trouble. As for QM requiring "the impossibility of distinguishing between material and non-material and even between being and non-being", few if any physicists would make such a claim. Of the handful that would, nearly all are Atheists like Lawrence Krauss who as we've seen, have no functional understanding of philosophy (or for that matter, even language and elementary logic) and are doing so for reasons that aren't even coherent, much less defensible. For more on all this, see Parts I and II of my series on Interpretations of Quantum Mechanics.

    That said, I would agree with Jaki that physics is to some extent descriptive of reality, and as such, cannot give us a completely unfiltered window to the metaphysical "thing in itself" underlying the universe. Bertrand Russell made similar statements as well. But to claim that it tells us absolutely nothing whatsoever about that reality amounts to a form of Aristotelian vitalism that is at best, little more than squid ink sophistry, and at worst positively ridiculous. With all due respect to Mactoul, and his continuing demand for answers to questions that have been answered repeatedly already notwithstanding, you'll be hard-pressed to find any scientists or analytical philosophers today who take such views seriously, and for damn good reasons.

    Best.

  67. Jonas says:

    Thank you for answering, Scott!

    Promptly quoted your part on the constants in the "discussion" where the question was raised to begin with. To my shame I must admit I didnt tell who the quote is from. This is mainly due to them not looking up the name, seeing that youre a christian, and concluding that therefore everything you say is biased nonsense. Luke Barnes knows this all too well.

    Have a great remaining week, everyone

  68. Scott Church says:

    Thanks @Jonas! No worries about not citing the quote to me. In my experience at least, Poisoning the Well fallacies are bread and butter for many, if not all Atheists, and I don't need the recognition badly enough to give them the opportunity hide behind them.

    Best. :-)

  69. Mactoul says:

    Scott Church,
    I take the liberty of answering your comments about the Jaki quote. How is Jaki wrong when he says about the philosophy of QM confounding material and non-material? Do you not yourself state that the wavefunction which is just a set of numbers is the fundamental building block of the matter?
    So, numbers --which I hope you would call non-material--build matter. This is what Jaki calls mathematical idealism.

    The science of QM implies nothing more than that wave function describes matter i.e. wave function of the hydrogen atom etc. But the physicist-philosophers such as Wheeler say that wave function itself is matter. Do you agree?

    "his continuing demand for answers to questions that have been answered repeatedly already notwithstanding"

    Answered but perhaps not to his satisfaction. Eg. your surprising assertion that the

    " the form or essential nature of the collection of things known as the universe is just the sum of the forms or essential natures of the things in the collection."

    I have pointed about problems involved in this statement and how this problem makes incomprehensible your primary assertion of the contingency of the universe--
    "The universe is contingent. Its essential nature, or form is separate from its existence. "

    I am curious actually as to what is the form or essential nature of the universe. Since I havent come across this concept previously.

  70. Claire Rousseau says:

    Hello gentleman. I find this blog fascinating., especially upon discovering that it is by scientists who are also fellow believers.
    I must confess, I am not a science buff at all. The Bible is my area of study. I do believe, however, that many scientists that have approached their field of study methodically & honestly, will at some point come to realize that virtually everything in the world around them points to a Creator. Creation itself points to a Creator.
    The irony is that most scientists (at least that I've engaged with) claim to be atheists. (I maintain that atheism is, in itself, a belief system). I've always wondered why that is.
    Forgive me, I do not mean to derail the conversation.

  71. Scott Church says:

    Hello @Claire,

    Thanks for the kind words! And my apologies for taking so long to reply. It's been a busy week for me. You are, of course right... all denials to the contrary notwithstanding, as I've shown here and elsewhere, Atheism indisputably is a belief system.

    Why do most of the scientists you've engaged with claim to be Atheists...? Well for starters, it's worth noting that while that may be true in your experience, generally it isn't. A number of studies have been done on this in recent years. Results vary of course, but globally, roughly half of all scientists believe in God and/or the supernatural. On a regional basis, those who identify with a religious tradition and/or claim to be religious vary from 30% in France, to as high as 94% in India (Ecklund et. al, 2016). The figure reported for the U.S. in that study was 39%. According to a 2009 survey of the American Association for the Advancement of Science conducted by Pew, nearly half of all scientists in that organization claimed a major religious affiliation. Less than half--41%--said they don't believe in God or the supernatural, and only 17% were willing to openly identify themselves Atheists by affiliation (Masci, 2009). What is true is that belief in God and/or the supernatural is less common overall in the scientific community than the general public. But the claim that all, or even most scientists are Atheists is an urban legend, which continues to have truck with the general public only because New Atheist talking heads like Richard Dawkins, Sam Harris, and others have made it a propaganda point, and few if any people ever bother to fact-check it.

    But that said, it is fair to ask why many scientists are drawn to Atheism, and why it's more common in the scientific community than the general public. In my experience at least, whether they're scientists or not, people are usually drawn to Atheism for at least one of three reasons, at least two of which aren't all that different from why others are drawn to God.

    First, for lack of a better term (and for reasons that will become apparent shortly) many scientists become Atheists because of what might be called the "Hammer" effect. As human beings, it's in our nature not only to strive for understanding of the world around us and give meaning to our lives but to want a key that unlocks all doors. If/when we find one, we tend to invest our hearts in it to the point of making a metaphysic out of it and deifying it. Ultimately, this is why idolatry is such an issue for us, & why so much of the Bible is devoted to dealing with it. Needless to say, few things have unlocked as many doors, endowed us with as many answers, or given us as much power as science. Add to that the adulation the general public bestows upon scientists for their accomplishments, and it's little wonder that scientists would view the tried and proven methods of their profession as not just a discipline but a metaphysical key to all mysteries--Scientism. But the trouble is... science isn't a metaphysic, it's a discipline, and confusion on that point can only lead to intellectual and spiritual train wrecks.

    As the Franciscan friar and thinker Richard Rohr has said... if your only tool is a hammer, pretty soon everything starts looking like a nail. One of the things I addressed in this essay is that while science has much to say about many theological questions, it's core issues and proofs are ultimately metaphysical, and as such don't lend themselves to scientific methodology. Once scientists decide that their professional methods are the only ones that have anything to say about truth--in other words, there's no such thing as a meaningful question that isn't a nail--it's only a matter of time until they deny the reality of screws, and break anything that can't be nailed together. As long as science is but one of many valuable tools in the box, and is used for what it's meant to address, it's a fountain of blessings. But when it's the only tool in the box, we end up with an intellectual and spiritual train wreck.

    Second, Atheism is attractive to many folks simply because it's fashionable. The details and history of this are far too involved to address here, but suffice to say that since the 60's the secular community has co-opted the words "reason" and "science", much the way the Religious Right has co-opted "moral" and "family values". In both cases, the association has less to do with any credible content in their arguments or living examples, than it does with gaslighting. Repeat something often enough, confidently enough, and in terms that resonate with society's pain, and like a Chinese water torture it will eventually wear an uncritical audience down to where questioning it is at best politically incorrect, and at worst irrational... whether it actually is or not. Most of the Atheists I personally know stubbornly insist that their views are "scientific" and mine "irrational", yet they refuse to read any peer-reviewed science whatsoever that I've put before them. And as near as I can tell, it never occurs to them that there's any inconsistency in that.

    Sadly, the Church is to blame for much of this. So many of our youth are raised with narrow-minded Fundamentalist oversimplifications that cannot survive the real world, and instead of equipping them for the challenges their faith will eventually have to confront, we teach them to retreat into bubbles of dogma, and lean on emotional experience rather than authentic struggles that require their minds as much as their hearts. And when that house of cards finally collapses, it's little wonder that secular fashions that deliver many of the same emotional drugs, without the real-world complications are more appealing (for more on this, see Mark Yoder's wonderful essay 10 Surprising Reasons Our Kids LEAVE Church).

    Finally, many folks become Atheists in response to the existential pain of their own experience. Some grew up in an overly harsh, emotionally and spiritually straight-jacketed church community that ran roughshod over the subtle landscape of their own humanity. As budding adolescents and young adults, they were forced to embrace a worldview that not only couldn't account for the world around them but couldn't account for their own humanity either, which like ours, was as complex and beautiful as it was broken. Eventually, their own survival required a rebellion, and when they did rebel there were plenty of fashionable secularists well-entrenched in the previous two reasons who were only too willing to toss them a life jacket. Again, much of this is the Church's fault.

    Others suffered great life tragedies, and couldn't come to terms with how a loving God could allow such things. One case in point was Charles Darwin. Darwin is often presented by the modern secular world as one who embraced science instead of religion, but in fact, his only degree is in... theology. He became an Atheist in 1851 when his daughter Annie died, more than five years before the Origin of Species was first published, not because science or reason compelled him to it, but because he was so wounded and disillusioned by her loss that he couldn't believe a loving God would've allowed such a thing.

    However one may respond to such tragedies, and whatever they may say about the nature of God, science had nothing whatsoever to do with these struggles. We certainly can understand how this kind of pain and loss might lead one away from God, or lead one to Him. But either way, when people reject God for reasons like these, it isn't because objective reason and evidence compel them to.

    In the end, I think Isaac Newton summed all of this up best when he said,

    "A little knowledge leads away from God, but much knowledge leads towards Him."

    Best. :-)

    REFERENCES

    Ecklund, E. H., Johnson, D. R., Scheitle, C. P., Matthews, K. R., & Lewis, S. W. (2016). Religion among scientists in international context: A new study of scientists in eight regions. Socius, 2, 2378023116664353. Online at journals.sagepub.com/doi/pdf/10.1177/2378023116664353. Accessed Oct. 6, 2017.

    Masci, D. (2009). Scientists and belief. Pew Research Centers Religion Public Life Project, Nov. 5, 2009. Online at http://www.pewforum.org/2009/11/05/scientists-and-belief/. Accessed Oct. 6, 2017.

  72. Colin Cooper says:

    Hi Scott,

    I just happened upon this blog the other day and have very much enjoyed reading your and Aron's very erudite posts on various scientific, philosophical and religious issues.

    It's nice to read informed, scientific perspectives that also take theistic understandings seriously.

    I agree with your assessment of the "multiverse" proponents in your footnotes to original post. To me - and I'm not a trained scientist or philosopher, it should be pointed out, just a humble lawyer - the multiverse seems like a good philosophical alternative to the "design" interpretation of the fine tuning problem but a "falsifiable scientific theory" it is not and nor is it ever likely to become one so far as I can tell, unless someone happens to spot tangible evidence of a "bubble collision" one day.

    In principle, the multiverse propsal looks to me almost indistinguishable from the "Deist God" in its appeal to a logically sound and elegant idea with great explanatory power but which is probably forever unobservable and devoid of any testable predictions.

    Simply put, if your best response to the argument that "the finely tuned and seemingly arbitrary initial conditions, laws and constants of the universe requires belief in the invisible, immaterial agency of a supreme being who exists outside the universe, to be adequately explained", is to posit the existence of something else outside our universe (a "multiverse") which is equally invisible to our observation and equally unprovable as a result...then you are essentially giving ground to the theist notion that, as it stands, there is no naturalistic explanation for fine tuning within the parameters of the observable universe (the only universe we know to exist and which we can study with scientific tools) and that answers need to be sought in unfalsifiable metaphysical realities beyond it.

    It is certainly a logical possibility that we live in an ever expanding megaverse of unlimited physical possibilities but unfortunately no one will ever be able to provide empirical evidence based upon scientifically robust tools of observation and falsifiable testing that is indicative of the "eternal inflationary multiverse" model - I think (and please correct this mathematically challenged mind of mine if I get this very wrong) on account of the particle horizon and the speed of light i.e. inflation causes the universe(s) to expand at a rate exceeding the speed of light, meaning that any envisaged bubble universes or "patches" in space are causally disconnected at practically infinite distances making communication between them or observation of them impossible for all eternity, among other factors.

    The "multiverse" is consequently a very poor answer to the theist fine tuning argument: seeking to refute an untestable but philosophically compelling notion ("the Deist God") with an equally untestable but philosophically compelling notion (an infinite multiverse of inflationary bubble universes each with randomly assigned physical laws and constants).

    Multiverse propositions are therefore the ultimate in circular reasoning for an atheist-naturalist minded person to employ in countering theistic arguments based on fine tuning - amounting to intellectual disarmament of the highest order.

    Also, something else merits attention here: an I wrong in thinking that the inflationary multiverse doesn't even get by "fine tuning" in the first instance since the "multiverse generator" and "cosmic inflation" itself (upon which the very possibility of a multiverse in space-time is predicated), would both require incredible fine tuning to operate and for the mathematical equations to work? The cure is thus worse than the disease and I believe this may have been noted by none other than Paul Steinhardt, one of the original pioneers of the "inflationary multiverse" hypothesis in the 1980s (who has since turned against it and is far more critical of the idea than I am, denying that it is even "explanatory").

  73. Aron Wall says:

    Great comment, Scott! And if anyone here hasn't yet read the article by St. Mark Yoder, they really should.

    (Although I don't think "gaslighting" is the best term for rhetoric that begins by fooling the person delivering the speech. I think I would prefer to use the term "sophistry". You know, the thing Socrates demolished by asking people annoying questions about what they really meant by various words!)

  74. Scott Church says:

    Thanks, Aron! I do agree that sophistry is a better label for the New Atheist community's most common arguments. What made me think of gaslighting here, was the fact that the core assumptions that most define New Atheism--"religion is incompatible with science", "faith is blind", etc.--have become no different than Trump's non-stop references to "crooked Hillary" last fall. Pelt the general public with sound bites like these often enough, and insistently enough, and eventually it won't matter whether they're true or not. They'll become dog-whistle phrases people just swallow because it's easier and more socially fashionable than due diligence. Especially if they touch an open wound (like, say, a bad childhood experience in Catholic school or a fundamentalist church), and those preaching them present themselves as "experts", and ambassadors for science and reason.

    I don't know if this was before your time or not, but in the 80's a comedy troupe called the Duck's Breath Mystery Theater had a regularly featured comedy show on NPR that captured this very dynamic in hilarious perfection... Ask Dr. Science. "Remember," the show always opened with, "he knows more than you do!", and "I have a Master's degree... in science!" Needless to say, all of Dr. Science's answers were entertainingly ludicrous. The humor of the show was based on the presumption that, just like real life, his audience would never question anything he said because he'd dazzled them with an aura of authority and repeated references to his "Master's degree"... in science!

    If you haven't heard it before, it's a riot! Ironically, New Atheism has drawn on precisely this sort of Chinese water torture sound-bite pelting and authoritarianism, and to great effect. In this respect at least, gaslighting comes to mind. :-)

    @Colin,

    Thank you for the kind words as well! Your thoughts pretty much hit the nail on the head, but here are some further quick comments...

    First, regarding your question about child universe particle horizons and the larger expansion rates in an inflationary multiverse, it is true that the resulting "bubble" universes will be separated from each other by enormously space-like distances that will preclude them from "contacting" each other. However, the phase transitions in the inflaton field that form these bubbles during "reheating" can propagate at rates faster than the local inflationary expansion rate in the bubble's neighborhood. As such, if two bubble universes were to form relatively close to each other it's possible for their boundaries (or "domain walls") to "bump" into each other before inflation separates them from each other permanently, and in theory at least, these bumps might leave some kind of detectable "bruise" on their respective cosmic microwave backgrounds. It's been suggested that if our universe experienced such a bump with a nearby child after the big bang, it might show up as an aberration in our CMB. Indeed, a few attempts have been made to find one, and a few rather hasty claims were made by folks who claimed to have found one. But as great as all this may sound on paper, in practice, it's exceedingly difficult to tie any detectable abnormalities in the CMB to something like this rather than any number of other explanations that don't require exotic, and otherwise unverifiable physics. The end result is that these claims have all evaporated, just like other attempts to observationally verify a multiverse.

    Second, regarding the problem of fine-tuning, it should be noted that by itself an inflationary multiverse is not enough to account for it. A nearly infinite number of universes with physics just like ours might explain why we exist in spite of all odds against that in such universes, but it would not explain fine-tuning in the physical parameters themselves (e.g. - the cosmological or fine structure constants) that make the formation of life even possible by any odds. To account for them we need eternal inflation in conjunction with string theory (or more properly, M-theory), and the so-called "string landscape". Without this landscape, there wouldn't be widely ranging variations in physical parameters from which child universes with combinations favorable to life like ours could be anthropically selected.

    The bottom line is that without M-theory, as well as eternal inflation, there is no alternative to fine-tuning and our existence that doesn't smack of a Creator. And as we've seen, not only is M-theory untestable, it won't be for the foreseeable future. As such, the entire string theory inflationary multiverse is every bit the article of faith that its Atheist proponents accuse religion of being (falsely), if not more so. :-)

  75. Aron Wall says:

    Scott & Colin,

    On the other hand, if the bubbles start far enough away from each other, then it is true that they will never be in casual contact.

    And since our own universe looks pretty isotropic, it really seems like there wasn't any collision with another bubble in our case...

  76. Scott Church says:

    Aron, yes. Interestingly, this reveals yet another side of the framework's indefeasibility. If our bubble universe formed close enough to another to bump into it and create a domain wall "bruise", then in principle we might be able to detect it (assuming, of course, that there's an adequate theoretical basis for uniquely discriminating what sort of CMB anisotropy or irregularity such a bruise would produce... which there isn't). However, if no such CMB irregularities are observed, the multiverse proponent can always claim that's because no bubble universes did form close enough to us to create one, so that outcome is to be expected as well. There always seem to be more than enough holes and moles to make up for every one that gets whacked.

    That said, the inflationary/string landscape is in many respects a beautiful framework and one that has led to much theoretical progress. But, it's also arcane enough to elevate its proponents to the status of omniscient wizards in the eyes of the general public. Add to that its indefeasibility, and the fact that its alternatives all smack of a Creator, and it's easy to see why so many Atheist physicists would rather dispense with testable predictions in science than let go of it. Which of course, tells us more about them than it does the universe. :-)

  77. Jonas says:

    Hello Scott and Aron,

    before coming to my (what seems to be weekly) question, a small apology to Aron: sorry to abuse your blog as a Q&A-hotline, I really try not to overdo it x) Of course the same applies to Scott, as hes the author of this article.

    Scott, as you argued for the impossibility of past-infinite causal chains somewhere (I believe..?), what would be your take on this short text from Facebook. Im not going to name the writer, but hes intellectualy honest, and as far as I can tell, smart. Which is why I take his thoughts seriously. As this is mostly a matter of philosophy of time, I would not feel secure enough to answer this.

    "I've tried on multiple occasions to prove that certain kinds of infinites are impossible, but came up empty-handed every time on closer analysis. Impossible means logically, contradictory. In a more physical sense, as per what exactly our universe is, how it works, things become significantly more complex, much more specific than mere logic. If you think you can derive a contradiction from an infinitude, then I'm genuinely interested. Below are a couple failed attempts I typed up."

    Here the failed attempts:

    "Various arguments has been proposed to prove that an infinite past is logically impossible. Here I’ll present two of them, and why they do not carry through. Note, some basic mathematics is required.

    Last Thursdayism

    Assumption (towards reductio ad absurdum): infinite temporal past
    Let’s enumerate past days up to and including last Wednesday as: {..., t, ..., -1, 0}
    That is, there exists a bijection among those past days (including Wednesday) and the non-positive integers.
    Now come Thursday.
    Observation: {..., t, ..., -1, 0} cannot accommodate Thursday
    Let’s re-enumerate the same past days but including Thursday as: {..., t, ..., -1, 0}
    That is, there exists a bijection among those past days (including Thursday) and the non-positive integers.
    Observation: {..., t, ..., -1, 0} can accommodate Thursday
    The two observations are contradictory.
    {..., t, ..., -1, 0} both cannot and can accommodate Thursday.
    Conclusion: the assumption is wrong, an infinite past is impossible
    Note, this argument could equally be applied to infinite causal chains, and nicely lends support to the Omphalos hypothesis (hence why I named it Last Thursdayism). Another thing to notice about the infinite set of integers: Any two numbers are separated by a number. And this number is also a member of the integers. The integers are closed under subtraction and addition. For the analogy with enumerating past days, this means any two events are separated by a number of days. Not infinite, but a particular number of (possibly fractional) days. That’s any two events. To some folk this is counter-intuitive, but, anyway, there you have it.
    The first observation is incorrect. Whether or not the set can accommodate Thursday (one more day), is not dependent on one specific bijection (the first selected), rather it is dependent on the existence of some (any such) bijection. A bijection also exists among {..., t, ..., -1, 0} and {..., t, ..., -1, 0, 1}, and the integers, for that matter.
    Therefore, the argument is not valid.

    The unnumbered now

    if the universe was temporally infinite, then there was no 1st moment
    if there was no 1st moment (but just some moment), then there was no 2nd moment
    if there was no 2nd moment (but just some other moment), then there was no 3rd moment
    ... and so on and so forth ...
    if there was no 2nd last moment, then there would be no now
    since now exists, we started out wrong, i.e. the universe is not temporally infinite
    Seems convincing at a glance?
    In short, the argument (merely) shows that, on an infinite temporal past, the now cannot have a definite, specific number, as per 1st, 2nd, 3rd, ..., now. Yet, we already knew this in case of an infinite temporal past, so, by implicitly assuming otherwise, the argument can be charged with petitio principii. That is, 6 is a non sequitur, and could be expressed more accurately as:
    5. if there was no 2nd last moment with an absolute number, then there would be no now with an absolute number
    6. since now exists, we started out wrong, i.e. any now does not have an absolute number
    Additionally, note that 1,2,3 refer to non-indexical “absolute” moments (1st, 2nd, 3rd), but 5 is indexical and contextual (2nd last, now), which is masked by 4. We already know from elsewhere (originating in linguistics) that such reasoning is problematic.

    Notes

    Hilbert’s Hotel and Shandy’s Diary, for example, are peripherally related. They’re what we call veridical paradoxes, and do not imply a contradiction, but they do show some counter-intuitive implications of infinites."

    The author then wrote down this quote, to further illustrate the point:

    "However, completing an infinite process is not a matter of starting at a particular time that just happens to be infinitely far to the past and then stopping in the present. It’s to have always been doing something and then stopping. This point is illustrated by a possibly apocryphal story attributed to the philosopher Ludwig Wittgenstein. Imagine meeting a woman in the street who says, “Five, one, four, one, dot, three! Finally finished!” When we ask what is finished, she tells us that she just finished counting down the infinite digits of pi backward. When we ask when she started, she tells us that she never started, she has always been doing it. The point of the story seems to be that impossibility of completing such an infinite process is an illusion created by our insistence that every process has a beginning.
    — Time: A Philosophical Introduction by James Harrington"

    Yet, said quote to me at least seems to rather illustrate the impossibility of such a situation, instead of arguing for its plausiblity. To be fair though, PoT is not something I have knowledge about, and Wittgenstein is quite the same to drop, so Im not willing to just write it off as confused.

    What are your thoughts on this everyone?

  78. Scott Church says:

    Hello @Jonas,

    Another interesting question! And no apologies needed Sir. Aron created Undivided Looking to be a resource for those seeking answers to questions involving physics, philosophy, and theology for those struggling to find answers, and I'm sure I speak for him as well in saying that we both take great joy in being able to help. Aron has always been there for me when I needed answers, and I take joy in doing so as well. Ergo, this is a Q&A-hotline! :-)

    My apologies for not responding sooner. It's been a busy week, and the weekend isn't looking much better. I haven't forgotten you though! I'll get back to you on all this soon as I have two minutes to rub together. Thanks for your patience, and have a great weekend! :-)

  79. Aron Wall says:

    Jonas,
    Your Facebook friend is quite correct: none of these misconceived arguments against the logical consistency of an infinite past are valid. Standard Cantorian set theory addresses all of these objections. An infinite past wouldn't need to have a "first" moment, anymore than an infinite future would have a "last" moment.

    I also don't think these arguments bear any resemblence to anything St. Scott or I have argued for. In this series I discussed some physics arguments that the universe began (and some that it didn't). I also argued that even if the universe had no beginning, it can still have aspects that would call out for a further explanation.

    In this blog post, Scott also mentioned the Scholastic metaphysical argument that there can't be an infinite regress of essentially ordered causes, but (at least the way St. Thomas Aquinas interpreted it), this argument has to do with simultaneous causation, not anything to do with whether the past is infinite.

    One can question either the physics or the metaphysics of these arguments, but at least none of them rest on crackpot mathematics the way that the ones your Facebook friend refuted do.

    As for the "woman in the street"; a woman isn't the sort of thing who plausibly has existed from all eternity, so it's no surprise if illustrating the story that way made it seem less plausible!

  80. Scott Church says:

    Hi @Jonas,

    Well, I survived my weekend and I'm back. Thanks again for your patience! Now, about your writer's thoughts regarding infinities...

    First and most importantly, as Aron pointed out, I never actually said past-infinite causal chains were impossible (at least, not in principle anyway). The real point is that whether they are or not is irrelevant. Either way, they cannot obviate the need for God. This guy is laboring mightily to get his hands on a metaphysical gun that has no bullets. To see why let's take a deeper look at the nature of causality.

    As we've seen, existing things--or more formally, substances--may possess powers to actualize potential changes in other substances (potencies) actively or passively. In the former case they're innate in the causal agent itself, and in the latter, acquired from another more causally primary substance. For instance, you have the active potency to wield a stick to roll a ball, whereas the stick has the passive potency to roll the ball only while you are imparting that potency to it by wielding it for that purpose. The stick cannot push the ball itself without you, or another wielding agent. But more can be said about how such causal activity manifests itself. In both cases, when one or more substances actualize some change in another, they impart to it something that is: a) Individually or collectively in them to begin with; and b) A reflection of some aspect of their essential nature, or form. Formally, we refer to these as the Principle of Proportionate Causality, and Agere Sequitur Esse (or, "action follows being"). [Here, and throughout this essay, I've been using the Thomistic or Aristotelian formal terminology, but regardless of how one chooses to identify and/or characterize them, the underlying principles involved remain essentially the same.]

    Now, this can happen directly or indirectly via multiple interactions, and the differences can be formally clarified as well. To borrow an example from Edward Feser, if I were to give you $20 I would be actively actualizing a potential of yours for increased net worth. There are a number of ways that could happen. If I have a $20 bill in my pocket, that bill can be said to be part of my immediate essence or form, and when I hand it to you, it directly becomes part of yours. In this case, I would be actualizing that potency formally. I could also write you a check or issue a money order, which you would then take to a bank where the $20 would become part of your account. In this case, I am acting in conjunction with a financial institution, including a bank teller or cash machine, to actualize that potency virtually. Finally, if I have access to U.S. Treasury printing equipment I could print you a $20 bill if I don't personally have one to give you. In this case, we say that I am actualizing the potency eminently on behalf of the Treasury. Notice that in all three cases your potential to be $20 richer is being imparted to you by one or more substances (me, the bank, the Treasury) that, individually or collectively, possess an actual $20 to pass along.

    These principles can be generalized to the rest of the universe as well, including properties not overtly manifested by the individual substances involved. Water, for instance, has many properties not manifested by hydrogen or oxygen alone. But their essential form is one that can combine to form water under the right environmental conditions. So, we can say that the collective actuality of hydrogen + oxygen + environmental temperature and pressure does possess the potency to actualize water. The bottom line is that actuality precedes potency and is more primary. Individually or collectively, substances cannot give to each other that which they do not in some sense actually possess. Essentially, this is yet another way to say that there's no such thing as magic.

    Now, with all that under out belts let's revisit my freight train example. As we've seen, freight cars have the potency to pull the cars behind them, but this potency is passive. They can only actualize it if they too are being pulled by the car/s in front of them. We can add freight cars to this train indefinitely. At every point, the resulting enumeration will have a bijection on the positive integers enumerating previous lengths of the train, out to infinity. But what will we then have...? [Wait for it...]

    An infinitely long chain of passive freight cars that cannot give to itself motive power it does not possess, and ain't goin' anywhere until an actively powered locomotive is hooked up to it! The same thing is true of a chain of freight cars hooked up in a loop.

    Now, that locomotive is itself a contingent combination of active and passive potencies as well, and we could chase this line of reason as far as we like, right down to its subatomic constituents and beyond. But sooner or later, we must either terminate this chain in something that is purely actual and inherently possesses all the given attributes or resort to magic. There is no third alternative. Notice also that this example is restricted to a single train of freight cars linked in a heirarchical causal chain in the present moment--I haven't said a word about anything else in existence, nor any temporal causal history. The cosmological argument doesn't need to consider either to get off the ground.

    Never mind the entire universe Jonas--even as it was last night much less over a potentially infinite past. Take me to the nearest trainyard today and I'll demonstrate God's existence to you here and now... and we can go get a drink afterward. :-)

    Even if I were to grant your guy his "Last Thursdayism" and "Unnumbered Now" arguments, he's still just as dead in the water for one simple reason. Both arguments are entirely mathematical, whereas the philosophical issues with a past-infinite universe are ontological. He claims that the former "could equally be applied to infinite causal chains" as well, but as we've seen, that's false. Read carefully between the lines of both arguments, and you'll see that they're based entirely on bijected enumerations along cardinal number lines. Considerations of active or passive causality are never involved.

    As Aron said, Cantorian set theory formally demonstrates how infinities work mathematically, and everyone on both sides of the debate (or nearly everyone anyhow) already knows that. The issue on the table isn't whether an internally consistent past-infinite mathematical model universe is possible--it's whether an actual physical one is. That's the whole point of Hilbert's Hotel. Hilbert wasn't making a mathematical argument with that example, he was demonstrating how real-world absurdities would result from any attempt to instantiate physical infinities in the actual universe itself. That question is actively debated and arguments have been given on both sides. But either way, regardless of the outcome of those discussions your guy's mathematical musings alone have little to do with it.

    And of course, as if the coffin needed yet another nail, we haven't even gotten yet to the fact that that for unrelated reasons addressed earlier and in my footnotes, it's quite likely the universe isn't past-eternal anyway, which if so, renders your guy's whole argument moot before it's even out of the gate.

    But all that said, if you're interested in wading deeper into the philosophical debate regarding past-temporal infinities, William Lane Craig is one of the thinkers who's been on the front lines of that discussion. In addition to his books, you can find some of his thoughts on the subject here and here.

    Best.

  81. Mactoul says:

    Scott Church,
    Are living things substances?
    Do they then have powers that are not reducible to powers of their components. For if ALL the processes in a living thing was reducible to chemical/atomic/ quark etc processes, then the living thing itself would be a agglomerate and not a true substance. I wonder if you agree.

    A related problem is how we end up in a world where discrete things exist, if the physics says that everything is basically wavefunction or fields over spacetime. A field is not a discrete thing.

  82. Jonas says:

    Scott,

    Thanks for the answer! I do have to wonder though; does William Lane Craig not hold to the A-theory of time? I thought physicists in special do not think too highly of that position.

    Jonas

  83. Scott Church says:

    Hi @Jonas,

    Yes, Craig is a supporter of A-Theory whereas most physicists are not. He has a few posts that discuss it at his Reasonable Faith website, but his most rigorous defense of it to date that I'm aware of is Time and Eternity. You might also have a look at Creation out of Nothing which he co-authored with Paul Copan. For more on why most physicists take issue with A-Theory, see Aron's God and Time series, Parts I, II, and III.

    Interestingly, Craig is also a supporter of divine simplicity--one of the hallmarks of Classical Theism, and very difficult to reconcile with a God who is neither timeless nor changeless. I haven't read either of these works far enough yet to know exactly how he makes that work.

    BTW, since I posted this essay last summer, Edward Feser has released his latest book Five Proofs for the Existence of God. This is one of his most in-depth expositions to date on the Aristotelian/Thomistic Classical Theism presented here, and I can't recommend it enough. The ideas I've covered are discussed at length in chapters 1, 4, 6, and 7.

    Best.

  84. Aron Wall says:

    Scott,
    It looks to me likw St. Craig doesn't accept Divine Simplicity.

  85. Scott Church says:

    Well, that's interesting! Here he responds to Dawkins' claim that God is complex by arguing that "as an unembodied mind, God is a remarkably simple entity". That's what I had in mind when I said he supported Divine Simplicity. But in this piece, he states that he only thinks God's cognition is simple. I'm not sure how either adds up to God not being simple... especially the former. But if I'm reading him right here, he rejects the "full-blown doctrine" on the grounds that God's nature can be described in terms of properties, where that word is being used in a univocal rather than an analogical sense. Feser discusses that at length in Five Proofs for the Existence of God (Chap. 7). I also now see that his response to Craig on theistic personalism here addresses divine simplicity as well.

  86. Scott Church says:

    Over the holidays I spent some time digging a little further into Sean Carroll's 2017 book The Big Picture and came to the conclusion that I hadn't done justice to his thoughts on the nature of causality and Aristotle's unmoved mover as well as I'd thought. So I revised and expanded my discussion of them, including an additional footnote and more sources. For those interested, a reread of that section (a little before the halfway point) might be worthwhile.

  87. Hi Scott,

    In one of your comment posts I found this:

    ‘But wait...! What about the "zero energy" universe? According to New Atheist physicist Lawrence Krauss, "nothing" has "no energy" so the universe originated from quantum fluctuations in a zero-energy vacuum.'

    I think this can be addressed in the following way:

    Let us admit that the universe has originated from nothing due to quantum fluctuations in a zero-energy-vacuum. As the universe has come from nothing, so initially space, time, matter and energy-all of them were zero. So the total space, total time, total matter and total energy of the universe should also always remain zero, because the universe is uncaused and therefore all its later evolution should be self-explanatory. Scientists have shown how the total matter-energy content of the universe always remains zero. But what about the total space-time? How does the total space-time of an ever-expanding universe always remain zero?

    When I have posed this question to some YouTube atheists, the usual reply was that although at present science does not have an answer to that question, but in future scientists will definitely find an appropriate explanation for that.

    So, let us wait and see.

  88. Scott Church says:

    Hello @Himangsu Sekhar pal,

    That's an excellent point... How does an obviously stable "nothing" spontaneously give rise to an evolving universe? As it happens, Aron discussed this in his post on Krauss' zero energy universe. But the real Achilles heel here is in the first sentence,

    Let us admit that the universe has originated from nothing due to quantum fluctuations in a zero-energy-vacuum...

    The problem with this is that quantum fluctuations in a zero-energy-vacuum are not "nothing"--They're quantum fluctuations in a zero-energy-vacuum.

    By definition, "nothing" is no thing--That is, the absence of any noun (person, place, or thing, including things like quantum vacuum field fluctuations). It's one thing to say that "nothing" has no energy. It's another thing altogether to say that it has an energy, and its energy is equal to zero. Only nouns can meaningfully have properties like energy. The second we try to assign an energy to "nothing" it becomes something. When it gets down to it, what Krauss and your Youtube atheists are really saying is that;

    Nothing is really something, and everything in existence created itself out of a thing which is really no thing and has no energy, which happens to be equal to zero joules...

    Today we cannot explain how this happened ("gee, ya think...?"), but some sweet day, Physics (peace be upon it) definitely will find an appropriate way to transform that statement into coherent English...

    Don't hold your breath. :-)

    One other thing to note... As I mentioned in my discussion of Carroll's arguments, scientists haven't actually shown how the total matter-energy content of the universe always remains zero. This would only be true if the universe happens to be spatiotemporally closed (and in that case, because of geometric constraints on any global definition of momentum/energy flux rather than anything to do with conservation laws). Apart from that, the energy of the universe is not even well-defined, much less obviously conserved. Krauss' attempt to appeal to the conservation of zero energy was based on a naively Newtonian argument that cannot be meaningfully applied to the universe as a whole.

    Best.

  89. Aron Wall says:

    Himangsu Sekhar pal,

    Energy is a number. Assuming this number is defined, it makes sense (possibly) to ask if that number can be zero. But spacetime is not a number. It is geometry. So I don't know what it would even mean to say that spacetime is zero.

    (Do you mean its spatial volume at one moment of time? That would be a number (assuming it is finite) but there is no rule in physics that the spatial volume of the universe is conserved...)

    But I also agree with the previous reply by St. Scott.

  90. Scott Church says:

    Aron,

    I trust he'll correct me if I'm mistaken, but I suspect what @Himangsu Sekhar pal meant by spacetime being zero was an initial state of a = 0--that is, no space-time--from which, it is said, fluctuations in some zero energy quantum field gave rise to an expanding space-time for which a(\tau) > 0 (\tau being comoving proper time. Krauss made statements to that effect himself in his comments after his debate with W.L. Craig when he said that by "nothing" he meant no space or time existed before a quantum fluctuation of some kind brought them into existence. He isn't asking atheists how an a = 0 space-time is conserved in an expanding universe. Rather, he's asking them how a quantum field with no space-time and a zero Hamiltonian can evolve into anything other than itself. As you've clearly shown elsewhere, that's a damn good question. :-)

  91. Hi Scott and Aron,

    I know very well that there is no conservation law for space-time, although there is a law of conservation for matter and energy. But there are the two theories of relativity. From SR we come to know that matter and energy are equivalent and GR says that space, time and matter are so interlinked that there cannot be any space and time without matter. I have also found this quote of Einstein somewhere in the internet:
    'When forced to summarize the general theory of relativity in one sentence: Time and space and gravitation have no separate existence from matter'.

    So, as per GR, there cannot be any space and time without matter. I think the reverse will also be true: there cannot be any matter without space and time.

    (This very nicely explains why God cannot be other than immaterial; God being spaceless and timeless God cannot be material.)

    My point is that if matter and energy are equivalent and if there cannot be any space and time without matter, then can we not say that there cannot be any space and time without energy?

    If this can be said, then supposing that the total energy of the universe always remains zero, should not the total space-time of the universe also always remain zero?

  92. Scott Church says:

    Hi @Himangsu Sekhar pal,

    Thanks for your reply! I think I understand what you've been saying a little better now. In answer to your question, in GR, space-time geometry is determined by its mass-energy, momentum, and pressure (specifically, by the stress–energy–momentum tensor T_{\mu \nu}. But nothing inherent in the theory or formalism strictly says that space-time cannot exist without matter or vice versa.

    When Einstein first developed GR he was intrigued by a conjecture known as Mach's principle, which states that local inertial frames are determined by the large-scale distribution of mass-energy in the universe. Its namesake, Ernst Mach, offered the analogy of riding a merry-go-round. When the merry-go-round is stationary, distant stars in the sky are as well and you feel no centripetal force. But when it's spinning, and you're on it, the stars are spinning from your reference frame and you do feel one. Mach believed this wasn't a coincidence and suggested that the centripetal force you experience is somehow related to the stars and other matter in the universe. Einstein has said that it was Mach's principle that first led him to GR, and the statement of his you quoted reflects that influence.

    But regardless of the role it played in inspiring him, the resulting formalism of the theory doesn't require it. It might be true (might be), but there are a number of valid solutions to the GR field equations that result in a universe with a space-time manifold containing no mass-energy, and others that do, but yield results which are inconsistent with the conjecture (including a famous one proposed by Kurt Godel based on a rotating universe). There are questions about these solutions as well so the matter is anything but settled. But either way, it must be remembered that Mach's principle is a conjecture only, and a controversial one that has a decidedly metaphysical component as well. Whether it's valid or not remains unclear, and from what I've seen, it's not supported by many physicists today. Ergo, as of this writing, nothing about GR requires a zero energy universe to be without a space-time manifold.

    Either way, none of this takes away from the from the force of the questions you put before your Youtube atheists or makes universes arising from fluctuating zero-energy quantum fields that are "no thing" anything other than the blathering incoherent nonsense they in fact are. :-)

    Best.

  93. Hi @Scott Church,

    I think zero-energy universe gives us one more reason for believing in the existence of God. If God is the creator, then he would be prior to the existence of space, time and matter. That is the reason we describe God as spaceless, timeless and immaterial. However one element is missing here: energy. God would be prior to the existence of energy also.

    If it is now true that energy can neither be created nor destroyed, then it would be true for God as well. This is because God being the law-giver we should not expect that God would break his own laws very frequently. So God would have to manage the entire creation event with zero energy.

    However if the universe is not created, then there is no reason as to why the universe as a whole cannot have total non-zero energy value. As some atheistic scientists claim that quantum laws were always there, so in a similar manner some sort of energy might have been always there. Universe would begin its life with that energy and it would also contain that much of energy as a whole.

    So, if the universe is created, then there would be at least one constraint as to why universe cannot have any energy. In the other case there would be no such constraint.

    However all the above are my speculations only and therefore they might be totally wrong.

  94. Pingback: Hvordan kan noen tro på Gud? - danieljoachim.org

  95. Pingback: Bayes’ Theorem VI: Does Theism Bear the Burden of Proof? | Veritas Christo et Ecclesiae

Leave a Reply

Your email address will not be published. Required fields are marked *

*

You may use these HTML tags and attributes: <a href="" title=""> <abbr title=""> <acronym title=""> <b> <blockquote cite=""> <cite> <code> <del datetime=""> <em> <i> <q cite=""> <strike> <strong>

help-hint.png
My comment policy, including help with leaving LaTeX equations. Place these between double dollar signs, for example: $$\hbar = 1.05 \times 10^{-34} \text{J s}$$. Avoid using > or < since these may be misinterpreted as html tags.
If your comment fails to appear do NOT submit it again.  Instead, email me so I can rescue it from the spam filter.  You can find my email by clicking on "webpage".